You are on page 1of 112

CIVIL LAW REVIEW II

PRACTICE QUESTIONS
AND
SUGGESTED ANSWERS
IN
CIVIL LAW REVIEW II

Obligations
Contracts
Sales
Lease
Credit Transactions
Agency
Trust
Partnership
Torts and Damages

Practice Questions and Suggested Answers 1


CIVIL LAW REVIEW II

INSTRUCTIONS

1. This is merely a compilation of practice questions and suggested answers. You are not supposed
to memorize these questions in lieu of reading cases and commentaries as preparation for your
exams.
2. As stated, these are merely SUGGESTED answers. You are free to disagree and have your own
answer or opinion.
3. This compilation is a mere supplementary material for your Civil Review II class. Do not rely on
this alone. Memorizing this by heart will not guaranty a passing grade for the class.

Practice Questions and Suggested Answers 2


CIVIL LAW REVIEW II

Table of Contents

Obligations: General Principles ..................................................................................................................... 4


Obligations .................................................................................................................................................. 13
Kinds of Obligations .................................................................................................................................... 17
Joint and Solidary Obligations (Computation) ............................................................................................ 22
Modes of Extinguishment ........................................................................................................................... 25
Contracts ..................................................................................................................................................... 30
Status of Contracts ...................................................................................................................................... 36
Sales ............................................................................................................................................................ 37
Status of Contracts ...................................................................................................................................... 58
Sales II ......................................................................................................................................................... 59
Maceda Law ................................................................................................................................................ 62
Status of Contracts of Sale .......................................................................................................................... 64
Remedies..................................................................................................................................................... 66
Extinguishment of Contract of Sale ............................................................................................................ 68
Lease ........................................................................................................................................................... 70
Credit Transactions: General Principles ...................................................................................................... 73
Credit Transactions ..................................................................................................................................... 79
Agency in General ....................................................................................................................................... 91
Agency ......................................................................................................................................................... 94
Partnership in General .............................................................................................................................. 100
Partnership................................................................................................................................................ 104
Torts and Damages in General .................................................................................................................. 106
Torts and Damages ................................................................................................................................... 109

Practice Questions and Suggested Answers 3


CIVIL LAW REVIEW II

OBLIGATIONS: GENERAL PRINCIPLES

TRUE OR FALSE

1. If the debtor obliged to do something fails to do it, an action for specific performance will
prosper.
a. FALSE. It will violate involuntary servitude. The remedy is the obligation may be done
by another at the expense of the debtor.

2. In conditional obligations, the obligation arises upon the happening of the condition.
a. FALSE. There are different kinds of conditional obligations. The condition may be
resolutory.

3. In case of non-compliance of obligations with a penal clause, the penalty shall substitute the
indemnity for damages and the payment of interest.
a. FALSE. The statement is not absolute; an exception is if there was an agreement.

4. The insolvency of one debtor will increase the liability of his co-debtors.
a. FALSE. This does not always apply. The liability will not increase if the obligation is
joint.

5. That obliged to deliver or do something (???) in delay only from the time the obligee judicially or
extra-judicially demands from them the fulfillment of their obligation.
a. FALSE. The statement is not absolute; an exception is when an obligation doesn’t need
demand but there is delay.
i. When stipulated by the parties, by provision of law, when time is the essence
or when the demand will be useless.

6. Proof of actual damages suffered by the creditor is not necessary in order that the penalty may
be demanded.
a. TRUE. Express provision, Art. 1228.

7. An obligation which is payable upon the death of Mr. X within five years from its execution is a
conditional obligation.
a. TRUE.

8. Solidarity may exist although the creditors and the debtors may not be bound in the same
manner and by the same periods and conditions.
a. TRUE. Art. 1207 provides that “there is a solidary liability only when the obligation
expressly so states, or when the law or the nature of the obligation requires
solidarity.”

Practice Questions and Suggested Answers 4


CIVIL LAW REVIEW II

9. When the fulfillment of the condition depends upon the sole will of the debtor, the obligation
shall be void.
a. FALSE. In resolutory conditional obligations, this does not apply.
b. A resolutory condition is the happening of the condition which extinguishes the
already existing rights and obligations.

10. In conjunctive obligations, the right of choice belongs to the debtor, unless it has been expressly
granted to the creditor.
a. FALSE. A conjunctive obligation is one where the debtor has to perform several
prestations and is only extinguished only by the performance of all of them.

11. It is a concept that derives from the principle that since mutual agreement can create a contract,
__________ by the parties can cause its extinguishment.
a. Mutual disagreement or decent.

12. Where an application for a loan of money is approved, there arises a perfected contract even
before the delivery of the object of the contract.
a. TRUE.

13. If the obligation has been substantially performed, the obligor shall recover as though there had
been a strict and complement fulfillment.
a. FALSE. Performance has to be in good faith. Art. 1234 provides: If the obligation has
been substantially performed in good faith, the obligor may recover as though there
had been a strict and complete fulfillment, less damages suffered by the obligee.

14. When the obligee accepts the performance, knowing its incompleteness or irregularity, the
obligation is deemed fully complied with.
a. FALSE. The acceptance must be without expression of any protest or objection. Art.
1235.

15. Payment by a judgment debtor of the wrong party does not extinguish judgment debt.
a. FALSE. This is only the general rule. The following are its exceptions:
i. Art. 1241
ii. Art. 1242
iii. Payment was delivered to the original creditor other than the new creditor.

16. Obligations requiring payment in foreign currency must be discharged in Philippine currency as
provided by RA No. 8183.
a. FALSE. RA 8183 allows the payment in different currency. In the absence of
agreement, payment should be made in Philippine Peso. What the law specifically
prohibits is the payment in currency other than legal tender. (Ponce v. CA)
b. RA 529

Practice Questions and Suggested Answers 5


CIVIL LAW REVIEW II

17. An agreement to pay a promissory note in dollars, while null and void under RA No. 529, does
not defeat a creditor’s claim.
a. TRUE. While an agreement to pay in foreign currency is declared null and void and of
no effect, the law does not defeat a creditor’s claim for payment because what the
law specifically prohibits is the payment in currency other than legal tender. (Ponce v.
CA)

18. A __________ implies that the check is drawn upon sufficient funds in the hands of the drawee,
that they have been set apart for its satisfaction and that they shall be so applied whenever the
check is presented for payment.
a. Certification.

19. The delivery of the promissory notes payable to order, or bills of exchange or other mercantile
documents produces the effect of payment.
a. FALSE. Under Art. 1249, delivery of such documents shall only produce the effect of
payment only when they have been cashed, or when thorough the fault of the
creditor, they have been impaired.

20. The creditor cannot be compelled to partially receive the prestation in which the obligation
consists. Neither may the debtor be required to make partial payments.
a. FALSE. As a general rule, the creditor may not be compelled to accept partial
fulfillment. The following are exceptions to this rule: (1) substantial compliance in
good faith, as provided under Art. 1234; (2) waiver, as provided under Art. 1235; or (3)
application of payments if the debts are equally onerous, as provided in Art. 1254(2).

21. The fact that the debtor had sufficient available funds on or before the grace period for the
payment of its obligation does not constitute proof of tender of payment.
a. TRUE. Tender of payment presupposes not only that the obligor is able, but more so in
the act of performing his obligation.

22. A certified personal check is not legal tender nor the currency stipulated and therefore cannot
constitute valid tender of payment.
a. FALSE. "Though a check is not legal tender, and a creditor may validly refuse to accept
it if tendered as payment, one who in fact accepted a fully funded check after the
debtor’s manifestation that it had been given to settle an obligation is estopped from
later on denouncing the efficacy of such tender of payment." (Far East Bank v. Diaz
Realty, August 23, 2001.)

23. The rule that in case an extraordinary inflation or deflation of the currency stipulated should
supervene, the value of the currency at the time of the establishment of the obligation shall be
the basis of payment is an applicable to obligations arising from tort.

Practice Questions and Suggested Answers 6


CIVIL LAW REVIEW II

a. FALSE. From the employment of the words “extraordinary inflation or deflation of the
currency stipulated”, it can be seen that the same envisages contractual obligations
where a specific currency is selected by the parties as the medium of payment. Thus,
this is inapplicable to obligations arising from torts and not from contracts. (Velasco v.
MERALCO)

24. Under the principle of dation in payment, the mere delivery of the mortgaged motor vehicle by
the mortgagor results in the transfer of ownership to the mortgagee even without the consent
of the latter.
a. FALSE.

25. There can be no consignation when there is no obligation to be extinguished.


a. TRUE. Consignation is the act of depositing the thing due with the court or judicial
authorities whenever the creditor cannot accept or refuses to accept payment and it
generally requires a prior tender of payment. (Likako v. Tedoro)
b. In order that consignation may be effective, the debtor must show that there was a
debt due. (De Leon v. Santiago Syjuco, Inc.)

26. __________ is the act of depositing the thing due with the court or judicial authorities whenever
the creditor cannot accept or refuses to accept payment.
a. Consignation.

27. Tender of payment may be extrajudicial.


a. FALSE. According to Jurado, tender of payment is, by nature, always extrajudicial.

28. Consignation of the amount required is not necessary to preserve the right to redeem.
a. TRUE. The right to redeem is a right, not an obligation. Thus, consignation of the
amount required is not necessary to preserve the right to redeem. (Immaculata v.
Navarro)

29. Difficulty of service authorizes release of obligor but does not authorize courts to modify or
revise the contract between the parties.
a. TRUE. Difficulty of service authorizes the release of the obligor but does not allow the
courts to remake, modify or revise the contract stipulated with the force of law, so as
to substitute its own terms for those covenanted by the parties themselves. (Occena
v. Jabson)

30. Performance is not excused by the fact that the contract turns out to be hard and
improvided(?), unprofitable or unexpectedly burdensome.
a. TRUE.

31. Consent is an essential requisite of obligation.

Practice Questions and Suggested Answers 7


CIVIL LAW REVIEW II

a. FALSE. It is not one of the essential elements because there can be an obligation even
without the consent of one of the parties.

32. A thing is indeterminate if it is not physically segregated from all others of the same class.
a. FALSE. This statement is not absolute. A thing may still be determinate even if not
physically segregated, as when the object is particularly designated.

33. In obligations with a term, the obligation arises upon the arrival of the period.
a. FALSE. The term or period has no effect upon the existence of the obligation, but only
their demandability or performance.

34. In obligations with a penal clause, the debtor may exempt himself from the performance of the
obligation by paying the penalty.
a. TRUE. The debtor cannot pay the penalty to relieve himself of the principal obligation,
unless that right is expressly granted to him. The payment of the penalty in lieu of the
principal obligation can be made only by express stipulation.

35. The insolvency of one debtor will increase the liability of his co-debtors.
a. FALSE. The insolvency of a debtor does not increase the responsibility of his co-
debtors, nor does it authorize a creditor to demand anything from his co-creditors.

36. The indivisibility of an obligation necessarily gives rise to solidarity.


a. TRUE. Express provision.

37. Whenever a period is designated in an obligation, it is for the benefit of both the creditor and
debtor.
a. FALSE. The rule under Art. 1196 is that the period is presumed to have been
established for the benefit of both creditor and debtor, unless from the same or other
circumstances it should appear that the period has been established in favor of one or
the other.

38. When the fulfillment of the obligation is dependent upon the sole will of the debtor, the
conditional obligation shall be void.
a. FALSE. Art. 1182 provides that when the fulfillment of the condition, not the
obligation, is dependent upon the sole will of the debtor, the conditional obligation
shall be void.
i. Even if condition, if condition is resolutory, obligation is not void.

39. In facultative obligations, the right of choice belongs to the debtor unless it has been expressly
granted to the creditor.

Practice Questions and Suggested Answers 8


CIVIL LAW REVIEW II

a. FALSE. The power of the debtor to make substitution is absolute.

40. Consignation, to be valid, generally requires a prior tender of payment.


a. TRUE. As a rule tender of payment is not necessary for a valid consignation; but it is
necessary if the consignation was made when the creditor to whom tender of
payment was made refused to accept it.

41. Consignation of the amount required is not necessary to preserve the right to redeem.
a. TRUE. Since the right to redeem is a right, not an obligation, consignation of the
amount required is not necessary to preserve the right to redeem. (Immaculata v.
Navarro)

42. Difficulty of service authorizes release of obligor but does not authorize courts to modify or
revise he contract between the parties.
a. TRUE. Difficulty of service authorizes the release of the obligor but does not allow the
courts to remake, modify or revise the contract stipulated with the force of law, so as
to substitute its own terms for those covenanted by the parties themselves. (Occena
v. Jabson)

43. It is a concept that derives from the principle that since mutual agreement can create a contract,
mutual disagreement can cause its extinguishment.
a. TRUE.

44. Where an application for a loan of money was approved, there arises a perfected contract
before the delivery of the object of the contract.
a. TRUE.

45. If the obligation has been substantially performed, the obligor shall recover as though there had
been a strict and complete fulfillment.
a. FALSE. Performance has to be in good faith and less damages suffered by the obligee.
Art. 1234 provides: If the obligation has been substantially performed in good faith,
the obligor may recover as though there had been a strict and complete fulfillment,
less damages suffered by the obligee.

46. When the obligee accepts the performance knowing its incompleteness or irregularity, the
obligation is deemed fully complied with.
a. FALSE. The acceptance has to be without expressing any protest or objection.

47. Payment by a judgment debtor to the wrong party does not extinguish judgment debt.
a. FALSE. This is only the general rule. The following are its exceptions:
i. Art. 1241
ii. Art. 1242

Practice Questions and Suggested Answers 9


CIVIL LAW REVIEW II

b. Payment was delivered to the original creditor other than the new creditor.

48. Obligations requiring payment in foreign currency must be discharged in Philippine currency as
provided by RA 8183.
a. FALSE. RA 8183 allows the payment in different currency. In the absence of
agreement, payment should be made in Philippine Peso. What the law specifically
prohibits is the payment in currency other than legal tender. (Ponce v. CA)
b. RA 529

49. An agreement to pay a promissory note in dollars, while null and void under RA 529 does not
defeat a creditor’s claim.
a. TRUE. While an agreement to pay in foreign currency is declared null and void and of
no effect, the law does not defeat a creditor’s claim for payment because what the
law specifically prohibits is the payment in currency other than legal tender. (Ponce v.
CA)

50. Consignation presupposes not only that the obligor is able, ready, and willing to pay, but mores
so in the act of performing his obligation.
a. TRUE.

51. “A proof that an act could have been done is no proof that it was actually done” supports the
concept known as tender of payment.
a. FALSE. Tender of payment presupposes not only that the obligor is able, but more so
in the act of performing his obligation.

52. A certified personal check is not the legal tender nor the currency contemplated, therefore
cannot constitute valid tender of payment.
a. FALSE. "Though a check is not legal tender, and a creditor may validly refuse to accept
it if tendered as payment, one who in fact accepted a fully funded check after the
debtor’s manifestation that it had been given to settle an obligation is estopped from
later on denouncing the efficacy of such tender of payment." (Far East Bank v. Diaz
Realty, August 23, 2001)

53. The rule that in case of extraordinary inflation or deflation of the currency stipulated should
supervene, the value of the currency at the time of the establishment of the obligation shall be
the basis of payment of obligations arising from law.
a. FALSE. From the employment of the words “extraordinary inflation or deflation of the
currency stipulated”, it can be seen that the same envisages contractual obligations
where a specific currency is selected by the parties as the medium of payment. Thus,
this is inapplicable to obligations arising from tots and not from contracts. (Velasco v.
MERALCO)

Practice Questions and Suggested Answers 10


CIVIL LAW REVIEW II

54. Under the principle of dation in payment, what is transferred is merely possession of the
property. (ownership)
a. FALSE. Dation in payment is the delivery and transmission of ownership of a thing by
the debtor to the creditor as an accepted equivalent of the performance of the
obligation. (Filinvest v. Phil. Acetylene)

55. There can be no dation in payment when there is no obligation extinguished.


a. TRUE. Dation in payment is the delivery and transmission of ownership of a thing by
the debtor to the creditor as an accepted equivalent of the performance of the
obligation. (Filinvest v. Phil. Acetylene)

56. Tender of payment may be extrajudicial.


a. FALSE.

57. Consignation is necessarily judicial.


a. TRUE. Art. 1258, “deposit to judicial authorities.”
b. It is judicial for it requires the filing of a complaint in court.

58. Demand is necessary to make an obligation due.


a. FALSE. Demand is not necessary to make an obligation due but to set the other party
in delay and to effectively toll applicable prescriptive periods.
b. As to when an obligation becomes due is not dependent on the existence of demand
in a pure obligation. Every obligation the performance of which does not depend on a
future or uncertain event, or a past event unknown to the parties, is demandable at
once. The same is true in obligations with resolutory condition or a resolutory period,
without prejudice to the effects of the happening of the condition or the arrival of the
period.
c. Also, in a reciprocal and simultaneous obligation, from the moment one of the parties
fulfil his obligation, the other party must be ready to comply with what is incumbent
upon him, or else delay will set in without the necessity of a demand.
i. However, in a reciprocal obligation which is not simultaneous, demand is
generally necessary, again not to make the obligation due but for delay to set
in upon the party who is not able to comply in the proper manner with what is
incumbent upon him.
d. Further, demand is made only upon the obligation becoming due. A demand made
upon an obligation that is not yet due and demandable will not set the other party in
delay. Thus, demand is futile.
e. There are other instances when demand is not necessary as to when the obligation
becomes due, as in:
i. when the law or the obligation expressly so declares, as for example, in a pure
obligation and those subject to a resolutory condition or period, which are due
and demandable at once;

Practice Questions and Suggested Answers 11


CIVIL LAW REVIEW II

ii. when from the nature and circumstances of the obligation it appears that the
designation of the time of fulfilment is the controlling motive of the
establishment of the contract; and
iii. when demand would be useless, as when the obligor has rendered it beyond
his power to perform.

59. The arrival of a period or the happening of a condition gives rise to an obligation.
a. FALSE. This statement is true only as regards obligations subject to a suspensive
condition or suspensive period, wherein the efficacy of the obligation is suspended
until the happening of the suspensive condition or the arrival of the suspensive
period.
b. First, there could be an obligation which is not dependent upon any future or
uncertain event. A pure obligation is demandable at once upon constitution. The
same is true in obligations subject to a resolutory condition or resolutory period,
without prejudice to the effects of the happening of the condition or the arrival of the
period, which extinguishes the obligation.

60. No person shall be responsible for those events which could not be foreseen, or which though
foreseen, were inevitable.
a. FALSE. This statement is generally true, but the civil code admits of exceptions.
b. First, in an obligation to give, the statement is true only when what is to be delivered
is a determinate thing. If it was a generic thing, the obligation will not necessarily be
extinguished by the happening of a fortuitous event, because of the principle that
genus never perishes.
c. The exceptions to this rule as provided by the civil code include, when the parties
expressly so stipulates the person to be responsible even for fortuitous events; or
when the law so provides; or when the nature of the obligation requires the
assumption of risks; or when the obligor is in delay upon the happening of the
fortuitous event; or when he has promised to deliver the same thing to two or more
persons who do not have the same interest, then the statement will not necessarily be
true anymore.

61. The value of the currency at the time of the constitution of the obligation shall be the basis of
payment in case of extraordinary inflation or deflation.
a. FALSE. This statement will only be true as regards obligations arising from contracts, in
case an extraordinary inflation or deflation of the currency stipulated should
supervene. The same provision already has an exception, where the parties agree on
the contrary. The provision speaks of parties and of agreement, thereby presupposing
that a contract was entered into between them. In obligations arising from law,
quasi-contracts, delicts and quasi-delicts, the applicable law or statutes should govern.

Practice Questions and Suggested Answers 12


CIVIL LAW REVIEW II

OBLIGATIONS

TRUE OR FALSE

1. Consent is an essential requisite of obligations.


a. FALSE. Not all obligations require consent; only contracts require consent.

2. A thing is indeterminate if it is not physically segregated from all others of the same class.
a. FALSE. This statement is not absolute. A thing may still be determinate even if not
physically segregated, as when the object is particularly designated.

3. In obligations with a term, the obligation arises upon the arrival of the period.
a. FALSE. The term or period has no effect upon the existence of the obligation, but only
their demandability or performance.
b. The effect of arrival of a period only goes into the demandability.

4. In obligations with a penal clause, the debtor cannot exempt himself from the performance of
the obligation by paying the penalty.
a. FALSE. This must be expressly granted to the debtor.

5. In obligations with a penal clause, the penalty shall substitute the indemnity for damages and
the payment of interest.
a. FALSE. The statement is not absolute; an exception is if there was an agreement.

6. The insolvency of one debtor will increase the liability of his co-debtors.
a. FALSE. This does not apply in obligations where the debtors are joint.

7. The indivisibility of an obligation does not necessarily give rise to solidarity.


a. TRUE. Express provision.

8. Whenever a period is designated in an obligation, it is for the benefit of both the creditor and
the debtor.
a. FALSE. This is a disputable presumption.
b. The rule under Art. 1196 is that the period is presumed to have been established for
the benefit of both creditor and debtor, unless from the same or other circumstances
it should appear that the period has been established in favor of one or the other.

9. When the fulfillment of the condition depends upon the sole will of the debtor, the conditional
obligation shall be void.
a. FALSE. This will only apply if the condition is suspensive. If the condition is resolutory,
the obligation shall be deemed valid.

Practice Questions and Suggested Answers 13


CIVIL LAW REVIEW II

10. In facultative obligations, the right of choice belongs to the debtor, unless it has been expressly
granted to the creditor.
a. FALSE. The choice only belongs to the debtor. The power of the debtor to make
substitution in facultative obligations is absolute.

MULTIPLE CHOICE QUESTIONS

1. A, B and C borrowed P36,000 from X. the debtors signed a promissory note on January 10, 2000,
promising to pay the creditor jointly and severally on or before July 10, 2000. How much can X
collect from C?
a. P18,000
b. P6,000
c. P12,000
d. P36,000
i. The debtors are bound solidarily, hence, any one of them may be made to pay
the whole amount of the obligation.
ii. “Jointly and severally” means solidary.

2. On January 1, 1999, A signs a promissory note and binds himself to pay X P100,000 plus 15% per
annum interest on June 30, 1999.
a. Before June 30, 1999, X can demand payment
b. IF on June 30, 1999, A is paying X, X can refuse the payment
c. Because the period is for the benefit of the debtor, A can compel the creditor X to
accept payment any date before June 30, 1999.
d. Because the period is for the benefit of the debtor and the creditor, X can refuse any
rendered payment before June 30, 1999.
i. The law presumes that the period is for the benefit of both debtor and
creditor. In this case, the period allows the debtor to raise money to pay the
obligation, while the creditor has interest.

3. Bertulfo and Claudio promised to deliver a particular car valued at P100,000 to Manuela on or
before September 15, 1999. September 15, 1999 came and upon demand by Manuela for
delivery from Bertulfo and Claudio, Bertulfo was willing to deliver but Claudio refused to deliver.
Thereafter, the car was destroyed. In this case:
a. An action for specific performance will lia against both Bertuflo and Claudio.
b. Both Bertulfo and Claudio shall be liable for P50,000 each with damages.
c. Bertulfo shall be liable for P50,000 without damages and Claudio shall be liable for
P50,000 and damages.
i. The obligation is joint and indivisible but it has become impossible to perform
due to the fault of one of the debtors. Thus, the obligation is converted to a
monetary obligation with damages for the debtor at fault.
d. An action for specific performance will lie against Claudio only because of his refusal.

Practice Questions and Suggested Answers 14


CIVIL LAW REVIEW II

4. In an obligation where only one prestation has been agreed upon, but to extinguish the
obligation, the debtor is allowed to render another prestation, the obligation is:
a. Disjunctive obligation
b. Facultative obligation
c. Obligation with penal clause
d. Alternative obligation
e. Joint obligation

5. An obligation ceases to be alternative and becomes a simple obligation:


a. When the debtor has already made a choice
b. When the creditor has already made a choice
c. When the choice of the debtor is consented to by the creditor
d. When the choice of the creditor is consented to by the debtor
e. None of the above
i. Correct answer is when the choice has been communicated to the other party.

6. It refers to a joint obligation:


a. One in which each debtor is liable for the entire obligation, and each creditor is entitled
to demand the whole obligation.
b. One in which either one of the parties is indispensable and the other is not necessary.
c. One in which the obligation of one is a resolutory condition of the obligation of the
other, the non0fulfillment of which entitles the other party to rescind the contract.
d. One in which each of the debtors is liable only for a proportionate part of the debt and
each creditor is entitled only for a proportionate part of the credit.

7. A and B are solidary debtor of X and Y, solidary creditors to the amount of P4,000. On the due
date, X renounced gratuitously in favor of A the entire obligation. Which of the following is
correct?
a. B shall give A P2,000
b. Y can still collect from B P2,000
c. Any of the two
d. None of the above
i. Renunciation of the entire obligation shall result in its extinguishment in
SOLIDARY OBLIGATIONS.

8. The creditor has the right to the fruits of the thing from:
a. The time the thing is delivered
b. The time the obligation to deliver the thing arises
c. The time the contract is perfected
d. The time the fruits are delivered

Practice Questions and Suggested Answers 15


CIVIL LAW REVIEW II

9. The issue of who has the right to choose may not occur in:
a. Conjunctive obligations
b. Facultative obligations
c. Alternative obligations
d. Both A and B
i. In conjunctive obligations, there is no issue regarding choice because all of the
prestations must be delivered for extinguishment.
ii. In facultative obligations, there is no issue regarding choice because the
debtor’s right of choice is absolute.
e. A, B and C
f. None of the above

10. D obliged himself to give C, object No. 1 valued P15,000; or object No. 2 valued P10,000; or
object 3 valued P5,000. All the objects were lost due to D’s fault in the order as stated.
a. D’s obligation is extinguished.
b. D’s obligation is to pay the value of object No.1 plus damages
c. C’s right to demand the value of any of the objects plus damages.
d. D’s obligation is to pay the value of object No. 3 plus damages.
i. In alternative obligations, the right of choice is presumed to belong to the
debtor.

Practice Questions and Suggested Answers 16


CIVIL LAW REVIEW II

KINDS OF OBLIGATIONS

TRUE OR FALSE

1. Every obligation whose performance does not depend upon a future and uncertain event, or
upon a past event unknown to the parties, is demandable at once.
a. FALSE. Art. 1179 – Every obligation whose performance does not depend upon a
future OR uncertain event, or upon a past event unknown to the parties, is
demandable at once.”
b. The statement is incomplete because it does not take into consideration obligations
with a period which depends upon a future and CERTAIN event.

2. Conditions which are contrary to public policy shall annul the obligation which depends upon
them.
a. TRUE. Art. 1183 – Impossible conditions, those contrary to good customs or public
policy, and those prohibited by law shall annul the obligation which depends upon
them.
b. While Art 1183 also talks about the obligation not being annulled if it is divisible, the
statement does not go into the divisibility or indivisibility of the same.

3. The condition shall be deemed fulfilled when the obligee voluntarily prevents its fulfillment.
a. FALSE. Art. 1186 – The condition shall be deemed fulfilled when the OBLIGOR
voluntarily prevents its fulfillment.
b. It is absurd for the obligee (creditor) to voluntarily prevent the fulfillment of the
condition. 

4. Obligations for whose fulfillment a day certain has been fixed, shall be demandable only when
that day comes.
a. TRUE. Art. 1193 – Obligations for whose fulfillment a day certain has been fixed, shall
be demandable at once. Obligations with a resolutory period take effect at once, but
terminate upon the arrival of the day certain.
b. The statement is absolutely true because it only pertains to obligations with a
suspensive period.

5. Whenever in an obligation a period is designated, it is deemed to have been established for the
benefit of both the creditor and the debtor.
a. FALSE.
b. Art. 1196 – Whenever in an obligation, a period is designated, it is PRESUMED to have
been established for the benefit of both the creditor and debtor, unless from the tenor
of the same or other circumstances, it should appear that the period has been
established in favor of one or of the other.

6. If the obligation does not fix a period, but from its nature and the circumstances it can be
inferred that a period was intended, the courts may fix the duration thereof.
a. TRUE. Art. 1197

Practice Questions and Suggested Answers 17


CIVIL LAW REVIEW II

7. The debtor shall lose every right to make use of the period when he does not furnish to the
creditor the guaranties or securities demanded by the latter after the obligation has been
contracted.
a. FALSE. Art. 1198(2) – The debtor shall lose every right to make use of the period when
he does not furnish to the creditor the guarantees or securities WHICH HE HAS
PROMISED.
b. Statement will only be true if debtor promised to furnish guaranties or securities.

8. A person alternatively bound by different prestations shall completely perform all of them.
a. FALSE. Art. 1199 – A person alternatively bound by different prestations shall
completely perform ONE of them.
b. A conjunctive obligation is one where the debtor has to perform several prestations; it
is extinguished only by the performance of ALL of them.

9. If through the debtor’s acts the creditor cannot make a choice according to the terms of the
obligation, the latter may rescind the contract with damages.
a. FALSE. Art. 1203 – If through the CREDITOR’S acts the debtor cannot make a choice
according to the terms of the obligation, the latter may rescind the contract with
damages.

10. The loss or deterioration of the thing intended as a substitute, through the negligence of the
obligor, does not render him liable.
a. TRUE. Art. 1206 – The loss or deterioration of the thing intended as a substitute,
through the negligence of the obligor, does not render him liable. But once the
substitution has been made, the obligor is liable for the loss of the substitute on
account of his delay, negligence, or fraud.
b. The statement is true because the first part of the provision used the word “intended”
which presupposes that the substitution HAS NOT YET BEEN MADE. The second
statement is not an exception to the first one because both contemplate different
scenarios.

MULTIPLE CHOICE QUESTIONS

1. D obliged himself to allow C the use of his car until C passes the CPA Examination. This is an
example of:
a. a pure obligation
b. an obligation with a suspensive condition
c. an obligation with a resolutory condition
d. an obligation with a suspensive period
e. an obligation with a resolutory period

 It is resolutory because the obligation is demandable at once. The statement also used the
word “until” which is an indicator that such is resolutory.
 It is a condition, not a period, because passing the CPA examination is a future and
UNCERTAIN event.

2. When the debtor binds himself to pay when his means permit him to do so, the obligation is:
a. a pure obligation

Practice Questions and Suggested Answers 18


CIVIL LAW REVIEW II

b. an obligation with a suspensive condition


c. an obligation with a resolutory condition
d. an obligation with a suspensive period
e. an obligation with a resolutory period

 Art. 1180 – When the debtor binds himself to pay when his means permit him to do so,
the obligation shall be deemed to be one with a period, subject to the provisions of Article
1197.
 The period is suspensive because the obligation will only be demandable upon the arrival
of the day when the debtor’s means permit him to do so.

3. The following obligations are immediately demandable, EXCEPT:


a. a pure obligation
b. obligation with a resolutory condition
c. obligation with a resolutory period
d. all of the above
e. none of the above

 Pure obligations are demandable at once. (Art. 1179)


 Obligations with a resolutory condition or period are also demandable at once. (Art. 1179
and Art. 1193)

4. In an obligation where only one prestation has been agreed upon, but to extinguish the
obligation the debtor is allowed to render another prestation, the obligation is a:
a. disjunctive obligation
b. obligation with a penal clause
c. alternative obligation
d. joint obligation
e. none of the above

 The statement refers to a facultative obligation.

5. An obligation ceases to be alternative and becomes a simple obligation:


a. when the debtor has already made a choice
b. when the creditor has already made a choice
c. when choice of the debtor is consented to by the creditor
d. when choice of the creditor is consented to by the debtor
e. none of the above

 Art. 1205 – The obligation shall cease to be alternative from the day when the selection
has been communicated to the debtor (or creditor, depending on who has the right to
choose).

6. “I promise to give A a car upon A’s passing the bar examinations.” The condition in this
obligation is:
a. resolutory
b. casual

Practice Questions and Suggested Answers 19


CIVIL LAW REVIEW II

c. possible
d. all of the above
e. none of the above

 A possible is one which is capable of fulfillment according to nature, law, public policy, or
good customs.
 It is not resolutory because it is not a certain event.
 The worst answer is CASUAL, because a casual condition is that which depends
EXCLUSIVELY upon chance or other factors, and NOT UPON THE WILL OF THE
CONTRACTING PARTIES. 

7. Which of the following conditional obligations is void?


a. When the happening of the suspensive condition is dependent upon the sole will of the
creditor.
b. When the happening of the suspensive condition is dependent upon the sole will of
the debtor.
c. When the happening of the suspensive condition is dependent upon chance.
d. When the happening of the resolutory condition is dependent upon the sole will of the
creditor.
e. When the happening of the resolutory condition is dependent upon the sole will of the
debtor.

 Art. 1182 – When the fulfillment of the condition depends upon the sole will of the
debtor, the conditional obligation shall be void.
 This applies only to conditions which are suspensive. It does not cover resolutory
conditions.

8. On January 1, 1999, A signed a promissory note binding himself to pay X P100,000 on or before
June 30, 1999.
a. Before June 30, 1999, A cannot be compelled to pay.
b. Before June 30, 1999, X can validly refuse an offer to pay.
c. Before June 30, 1999 while A cannot be compelled to pay, X also cannot be compelled
to accept payment.
d. Before June 30, 1999 while A can be compelled to pay, X cannot be compelled to accept
payment.
e. A, B, and C are correct.

 The period designated is for the benefit of the debtor only. Thus, he can oppose a
premature demand for payment, but may validly pay at any time before the period
expires.
 The phrase “on or before” is an indicator that the period has been established for the
benefit of the debtor.

9. The issue of who has the right to choose may NOT occur in:
a. conjunctive obligations
b. facultative obligations
c. alternative obligations

Practice Questions and Suggested Answers 20


CIVIL LAW REVIEW II

d. both a and b
e. a, b, and c

 In conjuctive obligations, the right to choose is a non-issue because all the prestations
have to be performed anyway.
 In facultative obligations, the right to choose is ALWAYS vested in the debtor while in
alternative obligations, the right may pertain even to the creditor or to a third person
when it has been expressly granted to either of them.

10. D obliged himself to give C, object no. 1 valued at P15,000; or object no. 2 valued at P10,000; or
object no. 3 valued at P5,000. All the objects were lost due to D’s fault in the order as stated. If C
has the right to choose:
a. D’s obligation is extinguished.
b. D’s obligation is to pay the value of object no. 1 plus damages.
c. C’s right to demand the value of any of the objects plus damages.
d. D’s obligation is to pay the value of object no. 3 plus damages.

 Art. 1205(3) – When the choice has been expressly given to the creditor, the obligation
shall cease to be alternative from the day when the selection has been communicated to
the debtor. … Until then, if all things are lost through the fault of the debtor, the choice by
the creditor shall fall upon the price of any one of them, also with indemnity for damages.

Practice Questions and Suggested Answers 21


CIVIL LAW REVIEW II

JOINT AND SOLIDARY OBLIGATIONS (computation)

1. in a joint obligation with V, W, X, Y and Z as debtors and A and B as creditors in the amount of
P900,000:
a. How much can A validly demand from V?
i. P90,000
ii. Pursuant to Art. 1208, there are a total of 10 obligations (5 debtors; 2
creditors)
b. How much can V recover from W if he paid the amount that A can compel him to pay?
i. None. V only paid his own share in the debt.
c. How much should A give to B if V paid him the amount that he can compel V to pay?
i. None. A only collected his share in the credit.

2. In a solidary obligation with V, W, X, Y and Z as debtors and A and B as creditors in the amount
of P900,000:
a. How much can A validly demand from X?
i. P900,000
ii. In solidary obligations, any one of the debtors may be compelled to pay for
the whole amount of the debt.
b. How much can X recover from W if he paid the amount that A can compel him to pay?
i. P180,000
ii. Unless there is a stipulation, debtors in a solidary obligation share equally in
paying the entire obligation.
c. How much should A give to B if X paid him the amount that he can compel X to pay?
i. P450,000
ii. Unless there is a stipulation, creditors in a solidary obligation share equally in
the credit.

3. In a passive solidarity with V, W, X, Y and Z as debtors and A and B as creditors in the amount of
P900,000:
a. How much can A validly demand from V?
i. P450,000
ii. In passive solidarity, solidarity exists among the debtors. A creditor can only
demand from any of the debtors his share in the credit.
b. How much can V recover from Z if he paid the amount that A can compel him to pay?
i. P90,000
c. How much should A give to B if V paid him the amount that he can compel V to pay?
i. None. A only collected his share in the credit.

4. In a solidary obligation with V, W, X, Y and Z as debtors and A and B as creditors in the amount
of P900,000 and V is insolvent:
a. How much can A validly demand from X?

Practice Questions and Suggested Answers 22


CIVIL LAW REVIEW II

i. P900,000
b. How much can X recover from Y if he paid the amount that A can compel him to pay?
i. P225,000
ii. Since the obligation is solidary, the insolvency of one of the debtors increases
the liability of the co-debtors.
c. How much should A give to B if X paid him the amount that he can compel X to pay?
i. P450,000

5. In a joint obligation with V, W, X, Y and Z as debtors and A and B as creditors in the amount of
P900,000 where V is insolvent:
a. How much can A validly demand X?
i. P90,000
ii. In joint obligations, the insolvency of one debtor does not increase the liability
of his co-debtors.
b. How much can X recover from Y if he paid the amount that A can compel him to pay?
i. None. X only paid for his share in the debt.
c. How much should A give to B if X paid him the amount that he can compel X to pay?
i. None. A only collected his share in credit.

6. In a solidary obligation with V, W, X, Y and Z as debtors and A and B as creditors in the amount
of P900,000, where V is insolvent and A remitted the share of Z with the consent of B:
a. How much can A validly demand from X?
i. P720,000
ii. The remission of A reduces the amount of the debt by the share of Z
(P180,000).
iii. Since the obligation is solidary, the insolvency of one debtor increases the
liability of the other co-debtors.
b. How much can X recover from Z if he paid the amount that A can compel him to pay?
i. P45,000
ii. Since the obligation is solidary, the insolvency of one debtor increases the
liability of the other co-debtors.
iii. This is Z’s share in covering the amount that V, their co-debtor, was unable to
pay due to his insolvency.
c. How much should A give to B if X paid him the amount that he can compel X to pay?
i. P360,000

7. In a solidary obligation with V, W, X, Y and Z as debtors and A and B as creditors in the amount
of P900,000, how much can A validly demand from W if V was a minor at the time the obligation
was constituted?
a. P720,000
b. Since the obligation is solidary, each debtor shall share equally in the debt, unless
there is a contrary stipulation. Thus, each co-debtor is liable for P180,000.

Practice Questions and Suggested Answers 23


CIVIL LAW REVIEW II

c. Minority of one of the co-debtors is a partial defense of the other co-debtors who are
not minors. Such partial defense may be invoked only insofar as the share of the
minor co-debtor.

8. In a solidary obligation with V, W, X, Y and Z as debtors and A and B as creditors in the amount
of P900,000, where A remitted the share of Z without the consent of B, how much can B validly
demand from X?
a. P720,000
b. Since the remission was without the consent of A, B can demand from him his whole
share of P450,000.
i. Hindi mababawasan ung share ni B because of the remission because he did
not consent to such.

Practice Questions and Suggested Answers 24


CIVIL LAW REVIEW II

MODES OF EXTINGUISHMENT

TRUE OR FALSE

1. Payment made by the debtor to a third person who is not the creditor shall never extinguish the
obligation.
a. FALSE.
b. Art. 1241 – Payment to a third person shall also be valid insofar as it has redounded to the
benefit of the creditor.
c. Art. 1242 – Payment made in good faith to any person in possession of the credit shall
release the debtor.
d. When creditor is incapacitated to receive payment or performance.

2. The delivery to and acceptance by the creditor of a cashier’s check produces the effect of
payment of the debtor’s obligation.
a. FALSE. Art. 1249 – The payment of debts in money shall be made in the currency
stipulated, and if it is not possible to deliver such currency, then in the currency which is
legal tender in the Philippines.
b. The delivery of promissory notes payable to order, or bills of exchange, or other
mercantile documents shall produce the effect of payment ONLY when they have been
cashed, or when through the fault of the creditor, they have been impaired.

3. Payment by cession does not always extinguish the whole obligation.


a. TRUE. Art. 1255 – This cession, unless there is a stipulation to the contrary, shall only
release the debtor from responsibility for the net proceeds of the thing assigned.

4. Legal subrogation requires the consent of the original parties and of the third person.
a. FALSE. Art. 1301 – Conventional subrogation of a third person requires the consent of the
original parties and of the third person.
b. Legal subrogation is subrogation by law. Unless there is a specific law providing for it, its
existence cannot be presumed. The consent of the parties has absolutely nothing to do
with it.

5. In novation, if the new obligation is void, the original one shall subsist, unless the parties
intended that the former relation should be extinguished in any event.
a. TRUE. Art. 1297

6. When the principal obligation is extinguished in consequence of a novation, accessory


obligations subsist.
a. FALSE. Art. 1296 – When the principal obligation is extinguished in consequence of a
novation, accessory obligations MAY subsist only insofar as they may benefit third persons
who did not give their consent.

7. Novation may be made even without the knowledge of the creditor.


a. FALSE. Art. 1293 – Novation may be made even without the knowledge or against the will
of the latter (debtor), but not without the consent of the creditor.
b. Consent of the creditor is ALWAYS necessary in novation.

Practice Questions and Suggested Answers 25


CIVIL LAW REVIEW II

8. Payment shall be made to one of the parties to the constitution of the obligation to extinguish
the obligation.
a. FALSE. Art. 1240 – Payment shall be made to the person in whose favor the obligation has
been constituted, or his successor in interest, or any person authorized to receive it.

9. Payment made in good faith to any person in possession of the evidence of credit shall release
the debtor.
a. FALSE. Art. 1242 – Payment made in good faith to any person in POSSESSION OF THE
CREDIT shall release the debtor.
b. Possession of credit IS DIFFERENT with possession of evidence of credit

10. Dation in payment is governed by the law of sales.


a. FALSE. Art. 1245 – Dation in payment, whereby the property is alienated to the creditor in
satisfaction of a DEBT IN MONEY, shall be governed by the law of sales.
b. If debt in money – law on sales; if debt is non-monetary – law on novation shall apply.

11. Legal compensation shall not be proper when one of the debts arises from a depositum.
a. TRUE. Art. 1287 – Compensation shall not be proper when one of the debts arises from a
depositum or from the obligations of a depositary or of a bailey in commodatum.
b. “Legal compensation is the true kind of compensation.” (URIBE) Thus, this provision
applies to all kinds of compensation, including legal compesation.

12. If a person should have against him several debts which are susceptible of compensation, the
rules on the application of payments shall apply to the order of the compensation.
a. TRUE. Art. 1289.

13. In partial compensation, no obligation is extinguished.


a. FALSE. Art. 1290 – When all the requisites mentioned in Art 1279 are present,
compensation takes place by operation of law, and extinguishes BOTH DEBTS TO THE
CONCURRENT AMOUNT, even though the creditors and debtors are not aware of the
compensation.

14. When one or both debts are rescissible or voidable, they may be compensated against each
other before they are judicially rescinded or avoided.
a. TRUE. Art. 1284.

15. In case an extraordinary inflation or deflation of the currency stipulated should supervene, the
value of the currency at the time of payment shall be the basis of payment.
a. FALSE. Art. 1250 – In case an extraordinary inflation or deflation of the currency stipulated
should supervene, the value of the currency at the time of the ESTABLISHMENT OF THE
OBLIGATION shall be the basis of payment, unless there is an agreement to the contrary.

16. If the debt produces interest, payment of the principal shall not be deemed to have been made
until the interests have been covered.
a. TRUE. Art. 1253.

17. An obligation which consists in the delivery of a determinate thing shall be extinguished if it
should be lost or destroyed due to a fortuitous event.

Practice Questions and Suggested Answers 26


CIVIL LAW REVIEW II

a. TRUE. Art. 1262 – An obligation which consists in the delivery of a determinate thing shall
be extinguished if it should be lost or destroyed without the fault of the debtor, and
before he has incurred in delay.
b. The delay is not a factor to consider in determining whether the obligation has been
extinguished or not. It is only relevant insofar as the remedies of the creditor against the
debtor are concerned.

18. Compensation shall take place in reciprocal obligations.


a. FALSE. Art. 1279 – In order that compensation may be proper, it is necessary:
(2) That both debts consist in a sum of money, or if the things due are
consumable, they be of the same kind, and also of the same quality if the latter
has been stated; …
b. In reciprocal obligations, there are different prestations, one is delivery of a thing and the
other is monetary.

19. The guarantor may not set up compensation as regards what the creditor may owe the principal
debtor
a. FALSE. Art. 1280 – Notwithstanding the provisions of the preceding article, the guarantor
MAY set up compensation as regards what the creditor may owe the principal debtor.

20. The obligation is extinguished from the time the characters of the creditor and debtor are
compensated in the same person.
a. FALSE. Art. 1275 – The obligation is extinguished from the time the characters of the
creditor and debtor are MERGED in the same person.

MULTIPLE CHOICE QUESTIONS

1. In three of the following cases, compensation shall NOT be proper. Which is the exception?
a. commodatum
b. gratuitous support
c. civil liability arising out of criminal offenses
d. bank deposit

2. When two persons in their own right are debtors and creditors of each other, there is:
a. confusion
b. compensation
c. consignation
d. novation

 Art. 1278 – Compensation shall take place when two persons, in their own right, are creditors
and debtors of each other.

3. A owes X P50,000 payable on or before June 30, 1999. S, who is not a party to the contract and
without the consent and against the will of A, paid X the P50,000 on April 1, 1999 when the
prevailing rate of interest was 12% per annum.
a. S can ask reimbursement from A in the amount of P50,000 plus 12% interest from April
1 to June 30, 1999.
b. S can ask reimbursement from A in the amount of P50,000.

Practice Questions and Suggested Answers 27


CIVIL LAW REVIEW II

c. S cannot ask reimbursement from A because the payment by S is without the consent
and against the will of A.
d. S can ask refund from X because the payment by S was against the will of A.

 Art. 1236 – Whoever pays for another may demand from the debtor what he has paid, except
that if he paid without the knowledge or against the will of the debtor, he can recover only
insofar as the payment has been beneficial to the debtor.
 Since the payment was made before June 30, there is no interest due to X.
 Also, because the payment was made without the consent and against the will of A, X can only
ask for reimbursement to the extent that A was benefited.

4. When an obligation is extinguished because of the passage of time, this is:


a. fulfillment of a resolutory condition
b. arrival of a resolutory period
c. novation
d. rescission

 A condition refers to an event, while a term or period refers to an interval of time. Since the
statement referred to the “passage of time,” the better answer is the arrival of a resolutory
period.

5. The following shall produce the effect of payment of debts:


a. delivery of check
b. tender of central bank notes
c. delivery of promissory note
d. dacion en pago

 The delivery of a check or of a promissory note will not produce the effect of payment UNLESS
they have been cashed or through the fault of the creditor, they have been impaired. (Art.
1249)
 The tender of central bank notes (money ) will also not produce the effect of payment if the
tender was not accepted.

6. Which of the following will NOT necessarily extinguish an obligation?


a. condonation
b. novation
c. death of the obligor
d. mutual dissent

 The death of the obligor will only extinguish an obligation if it is PURELY PERSONAL.

7. If a person pays the creditor without the knowledge or against the will of the debtor, the
following are true, EXCEPT:
a. He can recover only insofar as the payment has been beneficial to the debtor.
b. He cannot recover the amount from the creditor as payment of what is not due.
c. His remedy is only a simple personal action for reimbursement.
d. He can be subrogated to the rights of the creditor if the latter agrees.

Practice Questions and Suggested Answers 28


CIVIL LAW REVIEW II

 When payment by a third person is made without the knowledge of the debtor or against his
will, there will be no right of subrogation.

8. Payment made to a third person will NOT be valid if:


a. The payment redounded to the benefit of the creditor.
b. The payment was made to a person in possession of evidence of credit.
c. The payment was given to the original creditor after an assignment of the credit without
notice to the debtor.
d. The payment was made to an agent of the creditor who was authorized to receive
payment.

 Art. 1242

9. Which of the following will always extinguish an obligation?


a. tender of payment
b. insolvency of the debtor
c. death of the obligor
d. none of the above

 Tender of payment without acceptance from the creditor will not extinguish the obligation.
 The fact of insolvency of the debtor will also not extinguish the obligation. It is only upon the
declaration of the courts that the insolvency of the debtor may extinguish the obligation.
 The death of the obligor only extinguishes obligations which are purely personal.

10. X owes Y P40,000. With the consent of both parties, Z pays Y P20,000 which makes Y and Z
creditors of X to the amount of P20,000 each. X becomes bankrupt and has only P20,000 assets
left.
a. Z should get the P20,000.
b. Y and Z should divide the P20,000 equally.
c. X may choose whom to pay.
d. Y should get the P20,000.

 The original creditor is preferred over the new creditor in case the assets of the debtor is
insufficient to pay the debt.

Practice Questions and Suggested Answers 29


CIVIL LAW REVIEW II

CONTRACTS

TRUE OR FALSE

1. Contracts must bind both parties, its validity or termination cannot be left to the will of one of
them.
a. FALSE.
b. Its validity or compliance, not termination, cannot be left to the will of one of them.
(Art. 1308)

2. Unenforceable contracts are susceptible of ratification.


a. TRUE.
b. Unenforceable, unless they are ratified. (Art. 1403)

3. Contracts are perfected by mere consent.


a. FALSE.
b. This statement is only the general rule. An exception – real contracts such as deposit,
pledge and commodatum, are not perfected until the delivery of the object of the
obligation. (Art. 1316)

4. Ratification requires conformity of the contracting parties.


a. FALSE.
b. Ratification does not require the conformity of the contracting party who has no right
to bring the action for annulment. (Art. 1395)

5. Mistake as to the qualifications of one of the parties vitiates consent.


a. FALSE.
b. It will only vitiate consent when such qualification or identity have been the principal
cause of the contract. (Art. 1331)

6. Contracts take effect between their parties, assigns and heirs.


a. FALSE.
b. The statement is not absolute as when the rights and obligations arising from the
contract are not transmissible by their nature, or by stipulation, or by provision of law.
(Art. 1311)

7. A mere expression of an opinion does not signify fraud.


a. FALSE.
b. This is not absolute. It will only signify fraud when it is made by an expert and the
other party has relied on the expert’s special knowledge. (Art. 1341)

8. Incidental fraud invalidates a contract.


a. FALSE.
b. In order that fraud may make a contract voidable, it should be serious and should not
have been employed by both contracting parties. Incidental fraud only obligates the
person employing it to pay damages. (Art. 1344)

Practice Questions and Suggested Answers 30


CIVIL LAW REVIEW II

c. Dolo incident is fraud that does not have such a decisive influence and by itself cannot
cause the giving of consent; it refers only to some particular or accident of the
obligation.

9. In contracts of pure beneficence, the cause is the liberality of the benefactor.


a. TRUE.

10. The contract is void if the cause is not stated in the contract.
a. FALSE.
b. Although the cause is not stated, it is presumed that it exists and is lawful, unless the
debtor proves the contrary.

11. The contracting parties may establish such stipulations, clauses, terms and conditions as they
may deem convenient.
a. TRUE.

12. Acceptance by letter or telegram binds the offeror from the time the letter or telegram is sent.
a. FALSE.
b. It binds the offerer from the time it came to his knowledge. (Art. 1319(2))

13. An offer made through the agent is accepted from the time acceptance is communicated to the
agent.
a. TRUE.

14. There is violence when in order to wrest consent, serious or irresistible force is employed.
a. TRUE.

15. Failure to disclose facts constitutes fraud.


a. FALSE.
b. Failure to disclose facts will constitute fraud only when there is a duty to reveal them,
as when parties are bound by confidential relations. (Art. 1339)

16. Real contracts, such as mutuum, pledge and guaranty are not perfected until the delivery of the
object of the obligation.
a. FALSE.
b. Real contracts such as deposit, pledge and commodatum, not mutuum and guaranty,
are not perfected until the delivery of the object of the obligation. (Art. 1316)

17. Contracts of pure beneficence are contracts without cause.


a. FALSE.
b. In contracts of pure beneficence, the cause is the mere liberality of the benefactor.
(Art. 1350)

18. If the cause is not stated in the contract, it exists and is lawful.
a. FALSE.
b. It is presumed to be valid and existing, unless otherwise proven by the debtor.

Practice Questions and Suggested Answers 31


CIVIL LAW REVIEW II

19. A contract may be voidable even though there may have been no damage to the contracting
parties.
a. TRUE.

20. The action or defense for the declaration of the inexistence of a contract prescribes in 10 years.
a. FALSE.
b. The action or defense for the declaration of the inexistence of a contract does not
prescribe. (Art. 1410)

21. Acts and contracts which have for their objects the creation transmission, modification or
extinguishment of real rights over immovable property must be in a public instrument to be
valid and binding.
a. FALSE. Not for validity but for greater efficacy. Art. 1358 does not require such form
in order to validate the act or contract, but only to insure its efficacy. (Doliendo v.
Depino)

22. Contract of lease for 2 years is unenforceable if not in writing.


a. FALSE. An agreement for the leasing for a longer period than one year of real property
or of an interest therein, should be in writing, otherwise it will be unenforceable. (Art.
1403(2e)
b. Only if lease of real property.

23. An action for rescission of a contract entered into to defraud a creditor will prosper 4 years from
actual discovery of fraud.
a. FALSE. “From time of registration”.

24. Undue influence shall annul the obligation, although it may have been employed by a third
person who did not take part in the contract.
a. TRUE.

25. Rescission shall be only to the extent necessary to cover the damage caused by a contract in
fraud of a creditor.
a. TRUE.

26. A conveyance is presumed to be in fraud of creditor if made by a debtor after a suit is filed
against him while it is pending.
a. FALSE. Alienations by onerous title are also presumed fraudulent when made by
persons against whom some judgment has been issued. (Art. 1387)
b. conveyance must be onerous title

27. A contract which is void on its face cannot be the subject of reformation.
a. FALSE. The action for reformation of instruments presupposes that there is a valid
existing contract between the parties, and only the document does not correctly
express the terms of their true agreement.
b. When the real agreement is void, there can be no reformation.
c. only the subject of reformation is the real intention of the parties

Practice Questions and Suggested Answers 32


CIVIL LAW REVIEW II

28. Once a contract is perfected, though the same is not in the form required by law, the
contracting parties may compel each other to observe that form.
a. FALSE.
b. If the law requires a document or other special form, the contracting parties may
compel each other to observe that form, once the contract has been perfected. (Art.
1357)

29. Lesion or inadequacy of cause shall not invalidate a contract, unless there has been fraud,
mistake or undue influence.
a. FALSE.
b. Except in cases specified by law, lesion or inadequacy of shall not invalidate a contract,
unless there has been fraud, mistake or undue influence. (Art. 1355)

30. The cause in a lucrative contract is the liberality of the benefactor.


a. TRUE.

31. An offer may become ineffective upon insolvency of either party before acceptance is conveyed
even if the insolvency is not judicially declared.
a. TRUE.

32. A contract of sale entered into between spouses is void.


a. FALSE.

MULTIPLE CHOICE QUESTIONS

1. On February 11, 2003, Perfecto offered to sell his house and lot to Reynaldo for P1,500,000.00.
Perfecto told Reynaldo that he was giving Reynaldo up to February 28, 2003 to decide whether or
not to buy the house and lot. Reynaldo accepted the option but did not give anything to Perfecto to
support the option given to him. On May 20, 2003, Perfecto found another buyer who was ready to
buy the house and lot for P2,000,000.00. Perfecto wants to ask you whether he can still withdraw
the offer he made to Reynaldo. Decide.
a. Yes, Perfecto may withdraw the offer because he stands to gain an added profit of
P500,000.00 and this will be more than enough to pay damages to Reynaldo.
b. Yes, Perfecto may withdraw the offer by just informing Reynaldo of such fact.
i. When the offeror has allowed the offeree a certain period to accept, the offer
may be withdrawn at any time before acceptance by communicating such
withdrawal, except when the option is founded upon a consideration.
ii. In this case, the option was not founded upon any consideration.
c. No, Perfecto may not withdraw the offer because the option was accepted by Reynaldo
although Reynaldo did not give anything in support thereof.
d. No, Perfecto may not withdraw his offer until the lapse of the period given to Reynaldo
to exercise his option.

2. Elements that accompany certain contracts unless set aside or suppressed by the parties, are known
as:
a. Natural elements
i. These exist as part of the contract even if the parties do not provide for them
because the law creates them.

Practice Questions and Suggested Answers 33


CIVIL LAW REVIEW II

b. Essential elements
i. These are those that without such, there can be no contract. These are consent,
object and cause.
c. Accidental elements
i. These are those which are agreed upon by the parties and which cannot exist
without being stipulated.
d. Original elements

3. A contract where both parties are required to render reciprocal prestations, are known as:
a. Bilateral contract
b. Unilateral contract
c. Gratuitous contract
d. Commutative contract

4. Contracts take effect only between the contracting parties, their assigns and heirs except in cases
where obligations and rights arising from the contracts are not transmissible by their nature, or by
stipulation or by provision of law. This principle is known as:
a. Relativity of contracts
b. Mutuality of contracts
i. The contract must bind both contracting parties; its validity or compliance
cannot be left to the will of one of them. (Art. 1308)
c. Obligatory force of contracts
i. Obligations arising from contracts have the force of law between the
contracting paties and should be complied with in good faith. (Art. 1159)
d. Liberty of contracts

5. Obligations arising from contracts have the force of law between … (question incomplete)
a.
b.
c. ANSWER
d.

6. One of the following is not a real contract:


a. Pledge
b. Commodatum
c. Lease
i. The contract of lease may be of things or of work and service. (Art. 1642)
d. Mutuum

7. On May 1, 2002, S offered to sell a specific car to B for P500,000.00. B sent his letter of acceptance
to S on May 8, 2002. On May 10, 2002, however, S died in a vehicular accident and his secretary
received the letter or acceptance on May 12, 2002 unaware that S had already died.
a. The contract was perfected on May 8, 2002 when B sent his letter of acceptance.
i. Definitely wrong. Acceptance made by letter or telegram does not bind the
offeror until from the time it came to his knowledge.
b. The contract was perfected on May 12, 2002 when the secretary of S received the
letter.
i. Defnitely wrong. Secretary was not the offeror.

Practice Questions and Suggested Answers 34


CIVIL LAW REVIEW II

c. The contract was not perfected because the offer of S became ineffective when he
died.
i. Under the theory of manifestation, offer and acceptance takes effect only
from the time knowledge is acquired by the person to whom it is directed.
If during the intervening time, the offer or acceptance is extinguished by
death/insanity, such offer or acceptance has no more effect.
d. The contract was perfected on May 1, 2002 because the acceptance made by B on
May 8, 2002 retroacts to the date of the offer.

8. Mistake in three of the following will make a contract voidable. Which one will not?
a. Mistake as to the substance of the thing which is the object of the contract
b. Mistake as to the principal conditions which principally moved one or both parties to
enter into the contract.
c. Mistake as to the identity or qualifications of one of the parties, where identity or
qualifications have been the principal cause of the contract.
d. Simple mistake of account.

In order that mistake may invalidate consent, it should refer to the substance of the thing which is
the object of the contract, or those conditions which have principally moved one or both parties to
enter into the contract.

A simple mistake of account shall give rise to its correction. (Art. 1331)

9. It is the employment of serious or irresistible force to obtain consent.


a. Intimidation
i. There is intimidation when one of the contracting parties is compelled by a
reasonable and well-grounded fear of an imminent and grave evil upon his
person or property, or upon the person or property of his spouses,
descendants or ascendants, to give his consent.
b. Threat
c. Violence
d. Moral coercion

10. It is present when one of the contracting parties is compelled by a reasonable and well-
grounded fear of an imminent and grave evil upon his person or property or upon the person or
property of his spouse, descendants, and ascendants to obtain consent:
a. Violence
b. Physical coercion
c. Intimidation
d. Mistake

Practice Questions and Suggested Answers 35


CIVIL LAW REVIEW II

STATUS OF CONTRACTS

QUESTION ANSWER

1. Contacts undertaken in fraud of creditors when the latter cannot in any Rescissible
other manner collect the claims due them.

2. Sale of private agricultural land to a former Filipino citizen. Valid

3. The contract has a false cause. Void

4. Contract of sale where the consideration is in Chinese Yuan. Valid

5. Oral sale of immovable for P450. Unenforceable

6. Sale of large cattle contained in a private instrument. Void

7. Sale of a guardian of the property of the ward valued at P100,000 for Rescissible
P70,000.
8. Oral lease of a car for 3 years. Valid

9. Oral contract for the sale of a bag where the bag will be delivered after 18 Unenforceable
months and the payment shall be made upon delivery.

10. Absolute deed of sale where the price is not indicated. Valid

11. Contract where the signature of one of the contracting parties is forged. Void

12. One of the contracting parties is relatively incapacitated. Void

13. One of the contracting parties lacks juridical capacity. Void

14. One of the contracting parties is 20 years of age. Valid

15. A contact entered into by a person while insane during a lucid interval. Valid

16. Contract where one party is a minor and the other party is insane. Unenforceable

17. Contacts which refer to things under litigation if they have been entered Rescissible
into by the defendant without the knowledge and approval of the litigants
or of competent judicial authority.

18. … Voidable

19. Contacts where the consent is vitiated by mistake, violence, intimidation, Voidable
undue influence or fraud.

20. A contract entered into in representation of another without authority. Unenforceable

Practice Questions and Suggested Answers 36


CIVIL LAW REVIEW II

SALES

TRUE OR FALSE

1. A thing is determinate when it is particularly designated separating it from all others of the same
class.
a. TRUE.
i. Alternative answer: FALSE, because the statement implies that a thing will
only be determinate if it is particularly designated, excluding things which are
physically segregated.

2. In an absolute sale, ownership of the thing sold shall be transferred to the vendee upon the
delivery thereof.
a. FALSE.
b. The ownership of the thing sold shall be transferred to the vendee upon the actual or
constructive delivery thereof (Art. 1477), OR in any manner signifying an agreement
that possession is transferred from the vendor to the vendee (Art. 1496). (Baviera,
p.63)
c. In sale on approval, ownership passes to the buyer only when he signifies his approval
to the seller or when he does any other act adopting the transaction or when he
retains the goods without giving notice of rejection, or on the expiration of a
reasonable time, if no period has been fixed. (Art. 1502)
d. The parties may stipulate that ownership in the thing shall not pass to the purchaser
until he has fully paid the price. (Art. 1478)
e. Where there is a contract of sale of specific goods, the seller may, by the terms of the
contract, reserve the right of possession or ownership in the goods until certain
conditions have been fulfilled. (Art. 1503)

3. When adverse possession had been commenced before the sale but the prescriptive period is
completed after the transfer, the vendor shall not be liable for eviction.
a. TRUE. Art. 1550.

4. If two or more animals are sold together, whether for a lump sum or for a separate price for
each of them, the redhibitory effect of one shall only give rise to its redhibition, and not that of
the others.
a. FALSE.
b. The statement will only be true if it should NOT appear that the vendee would not
have purchased the sound animal or animals without the defective one. This will be
presumed when a team, yoke, pair, or set is bought, even if a separate price has been
fixed for each one of the animals composing the same. (Art. 1572)

5. For a sale of a thing to be valid, the thing must be determinate.


a. FALSE. The requisite that a thing be determinate is satisfied if at the time the contract
is entered into, the thing is capable of being made determinate without the necessity
of a new or further agreement between the parties. (Art. 1460, par. 2)

6. The price may be in a currency not legal tender here in the Philippines.

Practice Questions and Suggested Answers 37


CIVIL LAW REVIEW II

a. TRUE.

7. A vendor may be liable for eviction even if it is based on a right after the sale.
a. FALSE, the law provides “right PRIOR” to the sale, not after.
i. Alternative answer: TRUE. It is not only based on a right prior to the sale, but
also an act imputable to the vendor.

8. The vendee shall owe interest for the period between the delivery of the thing and the payment
of the price.
a. FALSE. The vendee shall only owe interest in the following instances:
i. Should it have been stipulated;
ii. Should the thing sold and delivered produce fruits or income; or
iii. Should he be in default, from the time of judicial or extrajudicial demand for
the payment of the price. (Art. 1589)

9. Possessory lien and the right of stoppage in transit cannot be exercised by the unpaid seller
simultaneously.
a. FALSE. In cases where there is partial delivery, the seller may exercise his right of
stoppage in transit for those goods which he has already parted with, and his right to
possessory lien on the remainder. (Art. 1528)

10. Goods are considered no longer in transit if the carrier refuses to deliver the goods to the buyer.
a. FALSE. It will only be considered no longer in transit if there was wrongful refusal to
deliver the goods to the buyer or his agent. (Art. 1531[3])

11. The contract is one of barter if the value of the thing to be delivered by one party exceeds the
value of the thing to be delivered by the other party.
a. FALSE. There is barter when it is manifested in the intention of the parties or when the
consideration is partly in money and partly in another thing when the value of the
thing given exceeds the amount of the money or its equivalent. (Art. 1468)

12. A contract of sale is essentially commutative.


a. TRUE. A commutative contract is when there is equivalency in the value of the
prestation to be performed by both parties. Normally, the thing sold would be equal
to the price paid by the other party. (Art. 2010)
b. Gaite v. Fonacier.

13. A sale of a right binds third persons if it is in a public instrument.


a. FALSE. The public instrument must also be registered before it can bind third persons.

14. In a contract to sell, the ownership of the thing automatically passes to the buyer upon the full
payment of the price.
a. FALSE. In a contract to sell, ownership is reserved by the seller despite the delivery to
the buyer. While a contract to sell is considered a special kind of conditional sale, it is
a peculiar kind of sale because despite the happening of the condition and actual
delivery, the buyer does not automatically acquire ownership.
b. If condition happens, the right of the buyer is to compel the seller to execute a final
deed of sale. So ownership does not automatically pass.

Practice Questions and Suggested Answers 38


CIVIL LAW REVIEW II

15. The option money may be considered as part of the price.


a. TRUE. Though it is option money, it can be considered as part of the price as long as it
is stipulated. Without stipulation, the option money cannot be considered as partial
payment because it is a consideration for the option and therefore not part of the
price.

16. A contract of sale is perfected upon the payment of the price.


a. FALSE. A contract of sale is perfected upon the meeting of the minds of the persons as
to the object and the price. (Art. 1475)

17. In a conditional sale, ownership passes to the buyer upon delivery.


a. FALSE. In conditional sales, conditions are imposed by the seller before ownership will
pass. Ownership automatically passes to the buyer from the moment the condition
happens. There is no need for another contract to be entered into.

18. A contract of sale where price is not money is void.


a. FALSE. The cause in a contract of sale may be money or its equivalent.

19. An assignment of credit is a consensual contract.


a. TRUE. An assignment of credit is perfected in accordance with the provisions of Art.
1475, where the law provides that a contract of sale is perfected at the moment there
is meeting of the minds. (Art. 1624)

20. Oral sale of a personal property with a market value of P700 is unenforceable.
a. FALSE. “Market value” is not the same as “price”.

21. When the sale is made through a public instrument, the execution thereof shall be equivalent to
the delivery of the thing which is the object of the contract.
a. FALSE. This will only apply if the contrary intention does not appear on the document.
(Art. 1498)

22. Where goods are shipped and by the bill of lading the goods are delivered to order of the buyer
or of his agent, ownership passes upon the delivery of the goods to the carrier.
a. FALSE. The effect is that seller reserves a right to the possession of the goods as
against the buyer.

23. The goods remain at the seller’s risk until the ownership therein is transferred to the buyer.
a. FALSE. There are instances when ownership has already transferred to the buyer but
the seller will still bear the risk; as when the seller is in delay.

24. Where delivery of the goods has been made to the buyer or to a bailee for the buyer, in
pursuance of the contract and the ownership in the goods has been retained by the seller
merely to secure performance by the buyer of his obligations under the contract, the goods are
at the buyer’s risk from the time of such delivery.
a. TRUE. Art. 1503.

Practice Questions and Suggested Answers 39


CIVIL LAW REVIEW II

25. Where goods are sold by a person who is not the owner thereof, and who does not sell them
under authority or with the consent of the owner, the buyer acquires no title to the goods.
a. FALSE. The buyer may acquire title provided that he bought it in good faith and for
value.

26. Where the seller delivers to the buyer a quantity of goods less than he contracted to sell, and
the buyer accepts or retains the goods as delivered, knowing that the seller is not going to
perform the contract in full, he must pay for them at their fair value.
a. FALSE. If the buyer accepts or retains the goods delivered with knowledge that the
seller is not going to perform the contract in full, he must pay for them at the contract
rate.
b. If however, the buyer has used or disposed of the goods delivered before he knows
that the seller is not going to perform his contract in full, the buyer shall not be liable
for more than the fair value to him of the goods so received. (Art. 1522)

27. Where in pursuance of a contract of sale, the seller is authorized or required to send the goods
to the buyer, delivery of the goods to a carrier, whether named by the buyer or not, for the
purpose of transmission to the buyer is deemed to be a delivery of the goods to the buyer.
a. FALSE. Art. 1523, par. 2.

28. In a lump sum sale of real estate, the vendor shall be obliged to deliver to the vendee, all that
may have been stated in the contract; but should this be not possible, the vendee may choose
between a proportional reduction of the price and the rescission of the contract.
a. FALSE. In the sale of real estate made for a lump sum, there is no increase or decrease
of price allowed although there be greater or lesser areas or number than that stated
in the contract. (Art. 1542)

29. If the same thing should have been sold to different vendees, the ownership shall be transferred
to the person who may have first taken possession thereof in good faith.
a. FALSE. Determine first if the thing sold was movable or immovable.
i. If movable, first who took possession in good faith has the better right.
ii. If immovable:
1. First to register the sale, in good faith; or
2. First to take possession, in good faith; or
3. Party who has oldest title, in good faith.

30. If the possessor of a movable lost or which the owner has been unlawfully deprived, has
acquired it in good faith at a public sale, the owner cannot obtain its return without reimbursing
the price paid therefor.
a. TRUE.

31. The owners of adjoining lands have the right of redemption when a piece of rural land, the area
of which does not exceed one hectare, is alienated.
a. FALSE, because the statement is incomplete; “unless the grantee does not own any
rural land.” (Art. 1621)

Practice Questions and Suggested Answers 40


CIVIL LAW REVIEW II

32. The fruits at the time of redemption will be protected between the redemptioner and the
vendee, giving the vendee the part corresponding to the time he possessed the land in the last
year, counted from the anniversary of the date of sale.
a. FALSE, because the statement is not applicable if there were fruits at the time of sale.
b. For conventional redemption. Can only exist in a sale with a right to repurchase.

33. In the warranty against eviction, the deprivation must be based on a right prior to the sale.
a. FALSE, because the deprivation can also be based on acts imputable to the vendor.

34. The contracting parties may increase, diminish, or suppress the obligation of the vendor to
warrant the object of the sale.
a. TRUE. Warranty is not an essential element of a contract of sale and may therefore be
increased, diminished, or suppressed by the agreement of the parties. (Art. 1548, par.
3)

35. The vendee need not appeal from the decision in order that the vendor may become liable for
eviction.
a. TRUE. Art. 1459.

36. When adverse possession had been commenced before the sale but the prescriptive period is
completed after the transfer, the vendor shall not be liable for eviction.
a. TRUE. Art. 1550.

37. If the property is sold for non-payment of taxes due as a consequence of which the vendee was
evicted, the vendor is liable for eviction.
a. FALSE, because the fact of non-payment of taxes should not be known to the vendee.
Also, the non-payment of taxes cannot be a basis for the vendor’s liability for eviction
because the taxes may become due after the sale. (Art. 1551)

38. Any stipulation exempting the vendor from the obligation to answer for eviction is void, if he
acted in bad faith.
a. TRUE. Art. 1553.

39. The right of pre-emption may be exercised by a co-owner.


a. FALSE, because the right of pre-emption is only exercised by adjoining owner of an
urban land.

40. The right of redemption of co-owners exclude that of adjoining owners.


a. TRUE. Art. 1623.

41. The “contract of sale with a right to repurchase” shall be presumed to be an equitable mortgage
when the vendee remains in possession as lessee or otherwise.
a. FALSE, because it should be the vendor that remains in possession as lessee or
otherwise. (Art. 1602, par. 2)

42. A co-owner of an undivided immovable who sells his share with a right to repurchase to a third
person who subsequently acquires the whole thereof, may be compelled by the latter to
redeem the whole property, if the former wishes to make use of the right of redemption.

Practice Questions and Suggested Answers 41


CIVIL LAW REVIEW II

a. TRUE. Art. 1620.

43. If the vendee made the waiver with knowledge of the risks of eviction and assumed the
consequences, the vendor shall not be liable.
a. TRUE. An equitable mortgage is a transaction that has all the requisites of a mortgage
but is not in the form of a mortgage.

44. The “contract of sale with a right to repurchase” shall be presumed to be an equitable mortgage
when it may be fairly inferred that the real intention of the parties is that transaction shall
secure the payment of a debt or the performance of any other obligation.
a. TRUE. Art. 1602.

45. The “contract of sale with a right to repurchase” shall be presumed to be an equitable mortgage
when the vendor binds himself to pay the capital gains tax.
a. FALSE, because paying capital gains tax, not being a tax on the thing, does not give rise
to the presumption that whoever is paying is the owner. (Art. 1602)
b. The presumption will only arise if the seller bound himself to pay the tax on the thing
not the capital gains tax. That would be the real property tax.

46. By the contract of sale one of the contracting parties transfer ownership over a determinate
thing and the other pays therefore a price certain in money or its equivalent.
a. FALSE, one of the contracting parties is obligated to transfer ownership AND deliver
the thing to the other. (Art. 1475)

47. The vendor must be the owner at the time of delivery for the sale to be valid.
a. FALSE, ownership is not an essential requisite.

48. The ownership of the thing sold shall be transferred to the vendee upon the delivery thereof.
a. FALSE, only applies to absolute sale.

49. The goods will be at the buyer’s risk after the delivery of the goods to the buyer.
a. FALSE, there are exceptions:
i. If there was a stipulation to the contrary (Lawyer’s Cooperative v. Tabora);
ii. There was delay in the delivery.

50. The ownership of the thing shall not pass to the purchaser until he has fully paid the price.
a. FALSE, there are exceptions:
i. Conditional sale
ii. Contract to sell
iii. Executory sale
iv. By stipulation

51. The sale may be valid even if the seller is not the owner of the thing sold.
a. TRUE, as when the sale is by auction or by persons with a statutory power to sell. (Art.
1505, par. 2)

52. A contract of sale may be valid even if the object is neither particularly designated nor physically
segregated from all others of the same class.

Practice Questions and Suggested Answers 42


CIVIL LAW REVIEW II

a. TRUE, the requisite that the thing be determinate is satisfied if at the time the
contract is entered into, the thing is capable of being made determinate without the
necessity of a new or further agreement between the parties. (Art. 1460, par. 2)

53. The price may be in a currency not legal tender here in the Philippines.
a. TRUE.

54. Service may be the subject matter of a valid sale.


a. FALSE, things and rights only.

55. The sale of an immovable for a price less than P500 shall be enforceable even if not in writing.
a. FALSE, sale of an immovable must be in writing regardless of price.

56. In a contract of sale, the seller will have the obligation to take care of the thing sold with the
diligence of a good father of a family.
a. FALSE, this is not absolute. This will not apply if:
i. There was a stipulation on the contrary;
ii. Traditio brevi manu
iii. Sale of a generic thing.

57. When goods are delivered to the buyer on satisfaction, the ownership therein may pass to the
buyer even if the buyer does not signify his approval or acceptance to the seller.
a. TRUE, this may occur when the buyer:
i. Does any other act adopting the transaction;
ii. Retains the goods without giving notice of rejection, then if a time has been
fixed for the return of the goods, on the expiration of time, and if no time has
been fixed, on the expiration of a reasonable time.

58. The risk of loss shall be borne by the owner.


a. FALSE, there are exceptions:
i. If there was a stipulation to the contrary (Lawyer’s Cooperative v. Tabora);
ii. Title was reserved by the seller to secure the payment of the price by the
buyer;
iii. There was delay in the delivery.

59. Dation in payment, whereby property is alienated to the creditor in satisfaction of a debt, shall
be governed by the law of sales.
a. FALSE, dation in payment will only be governed by the law on sales if the debt was in
money.

60. A contract for the delivery at a certain price of an article which the vendor in the ordinary course
of business manufactures or procures for the general market, if the same is on hand, is a
contract for a piece of work.
a. FALSE, the statement pertains to a contract of sale.
b. It is a contract for a piece of work if the seller does not normally produce or
manufacture the thing for the general market. (Art. 1467)

61. A contract of sale with things having a potential existence as object may be a valid sale.

Practice Questions and Suggested Answers 43


CIVIL LAW REVIEW II

a. TRUE. Art 1461.

62. The goods remain at the seller’s risk until the ownership therein is transferred to the buyer, but
when the ownership therein is transferred to the buyer the goods are at the buyer’s risk
whether actual delivery has been made or not.
a. FALSE, there are exceptions:
i. Unless otherwise agreed upon;
ii. Title was reserved by the seller to secure the payment of the price by the
buyer;
iii. If there was delay in the delivery, person who was in delay will bear the loss.

63. In a contract to sell, ownership passes to the buyer upon full payment of the price.
a. FALSE, seller still needs to transfer ownership and deliver.

64. If the possessor of a movable lost or which the owner has been unlawfully deprived has
acquired it in a public sale, the owner cannot obtain its return without reimbursing the price
paid therefor.
a. FALSE, the owner can recover the thing lost without reimbursement if the buyer was
in bad faith. (Art. 559)

65. Where goods are sold by a person who is not the owner thereof, and who does not sell them
under authority or with the consent of the owner, the buyer acquires no better title to the
goods than the seller had.
a. FALSE, this will be true UNLESS the owner of the goods is by his conduct, precluded
from denying the seller’s authority to sell. (Art. 1505)

66. When goods are delivered to the buyer on “sale or return”, ownership is transferred when he
signifies his approval or accepts to the seller or does any other act adopting the transaction.
a. FALSE, this applies to a “sale on approval or satisfaction”.
b. In a “sale or return”, ownership is transferred to the buyer upon delivery but he may
revest the ownership in the seller by returning or tendering the goods within the time
fixed in the contract or within a reasonable time. (Art. 1502)

67. If a movable property is sold to two or more persons, ownership shall belong to the person who
may have first taken possession thereof.
a. FALSE, it must be a possession in good faith. (Art. 1544)

68. Contracts of sale shall be obligatory, in whatever form they may have been entered into,
provided all the essential requisites for their validity are present.
a. FALSE, parties may reciprocally demand performance, subject to the provisions of the
law governing the form of contracts. (Art. 1475)

69. The authority of the agent to sell a piece of land must be in writing; otherwise, the agency is
void.
a. FALSE, sale, not the agency, is void. (Art. 1874)

70. The expenses for registration and execution of the sale are borne by the vendee unless
otherwise stipulated.

Practice Questions and Suggested Answers 44


CIVIL LAW REVIEW II

a. FALSE, expenses are borne by the vendor, not the vendee, unless otherwise
stipulated. (Art. 1487)

71. If the consideration of the contract consists partly in money and partly in another thing, it shall
be considered a barter if the value of the thing given a part of the consideration exceeds the
amount of the money or its equivalent.
a. FALSE, consider the intent of parties first. (Art. 1468)

72. There may be a contract of sale of goods whose acquisition by the seller depends upon
contingency which may or may not happen.
a. TRUE. Art. 1462.

73. The sale of a vain hope or expectancy is void.


a. TRUE. Art. 1461.

74. If the price is simulated, the sale is void, but the act may be shown to have been in reality a
donation or some other act or contract.
a. TRUE. Art. 1471.

75. A contract of sale entered into between a husband and wife is void.
a. FALSE, there are exceptions:
i. If the spouses have a marriage settlement of complete separation of property;
ii. If there is a judicial separation of property. (Art. 1490)

76. A thing is determinate when it is particularly designated.


a. TRUE.

77. For a sale to be valid, the object must be determinate.


a. FALSE. It is sufficient that it can be made determinate at the time the contract is
entered into without the necessity of a new and further agreement between the
parties.

78. A sale is valid even if the consideration to be paid is in Chinese Yuan.


a. TRUE. A price certain may be in MONEY or its equivalent. Money does not have to be
legal tender.

79. Goods are no longer in transit if the carrier or other bailee refuses to deliver the goods to the
buyer or his agent in that behalf.
a. FALSE. It has to be WRONGFUL refusal. (Art 1531)

80. If two or more animals are sold together, whether for a lump sum or for a separate price for
each of them, the redhibitory defect of one shall only give rise to its redhibition, and not that of
the others.
a. FALSE. There may be a redhibition of all animals when it should appear that the
vendee would not have purchased the sound animal or animals without the defective
one. (Art 1572)

Practice Questions and Suggested Answers 45


CIVIL LAW REVIEW II

81. When adverse possession had been commenced before the sale but the prescriptive period is
completed after the transfer, the vendor shall not be liable for eviction.
a. TRUE. Art. 1550.

82. Sellers are liable for breach of warranty.


a. FALSE. Sellers are not always liable for breach of warranty, as when they have
complied with their obligation, and in accordance with their warranties. Further,
warranties may be waived.

83. The vendee shall owe interest for the period between the delivery of the thing and payment of
the price.
a. FALSE. This is only true in the following three cases:
i. should it be stipulated;
ii. should the thing sold and delivered produce fruits and income;
iii. should he be in default, from the time of judicial or extrajudicial demand for
the payment of the price. (Art 1589)

84. Seller may exercise possessory lien and right of stoppage in transitu at the ANY time.
a. FALSE. The right to retain possessory lien presupposes that the seller must still have
possession of the goods. In the right of stoppage in transitu, it is necessary that the
seller has parted with the thing sold and the buyer must be insolvent.

85. Di ko maalala yung isa pa. Something to do with ownership vesting with the buyer at perfection?
a. FALSE.

86. The unpaid seller loses his lien on the goods when he delivers the goods to a carrier or other
bailee for the purpose of transmission to the buyer.
a. FALSE, because there are exceptions to this rule:
i. Seller may reserve ownership to the goods or the right to possession thereof.
(Art. 1529)

87. The seller’s possessory lien or right of stoppage in transitu is not affected by any sale or
disposition of the goods made by the buyer.
a. FALSE, the seller may lose his lien when he assented to the sale or dispositon; or the
goods are covered by a negotiable instrument. (Art. 1535)

88. Notice need not to be given to the original buyer of the intention to resell the goods for the
validity of the resale.
a. TRUE. Art. 1533

89. The right of redemption of co-owners exclude that of adjoining owners.


a. TRUE. Art. 1623

90. If a small piece of rural land which was bought for speculation has been resold, the owner of the
adjoining has a right of redemption at a reasonable price.
a. FALSE, because the statement pertains to rural land when it should be urban land.

Practice Questions and Suggested Answers 46


CIVIL LAW REVIEW II

91. The apparent vendor may ask for the reformation of the instrument in the contracts presumed
to be a sale with a right to repurchase.
a. FALSE, because what should be presumed is an equitable mortgage instead of a sale
with a right to repurchase.

92. If refusal to accept delivery is justified, title to the goods does not pass on to him.
a. TRUE.

93. The remedies under the Maceda Law are alternative.


a. FALSE, remedies under the Maceda Law are cumulative, not alternative. It is the
remedies under the Recto Law that are considered to be alternative.

94. Conventional redemption shall take place when the vendor reserves the right to repurchase the
thing.
a. FALSE, because the word “shall” implies that conventional redemption shall only take
place when the vendor reserves the right to repurchase.

95. In sale with right to repurchase, the vendee is subrogated to the vendor’s rights and actions.
a. TRUE.

MULTIPLE CHOICE QUESTIONS

1. When the goods are delivered to the buyer, the ownership passes to the buyer in:
a. Sale on approval
i. Ownership passes to the buyer when (1) he signifies his acceptance or does
any other act adopting the transaction; (2) retains the goods without giving
notice of rejection, then if a time has been fixed for the return of the goods,
on the expiration of such time, and if no time has been fixed, on the expiration
of a reasonable time. (Art. 1502)
b. Sale on satisfaction
i. Same with sale on approval.
c. Sale on trial
i. Same with sale on approval.
d. Sale or return
i. In a sale or return, the buyer acquires ownership upon delivery of the thing
sold and may revest the ownership to the seller within the time fixed in the
contract or within a reasonable time. (Art. 1502)
e. Contract to sell
i. It is considered a special kind of conditional sale, it is a peculiar kind of sale
because despite the happening of the condition and actual delivery, the buyer
does not automatically acquire ownership. If condition happens, the right of
the buyer is to compel the seller to execute a final deed of sale. So ownership
does not automatically pass.

2. It is an affirmation of fact or any promise by the seller relating to the thing which has a natural
tendency to induce the buyer to purchase the same, relying on such promise or affirmation.
a. Condition
b. False representation

Practice Questions and Suggested Answers 47


CIVIL LAW REVIEW II

c. Warranty
i. Art. 1546
d. Seller’s talk

3. S makes an offer to B on January 1, 1996. B makes known his acceptance in a letter sent on
January 2, and received by the secretary of S on January 10. Meantime, on January 5, S died.
a. There is no contract.
i. An offer becomes ineffective upon the death of either party before acceptance
is conveyed. (Art. 1323)
ii. The offer is deemed accepted once the seller has knowledge of the buyer’s
acceptance.
b. The contract is voidable because a party is insane.
c. There is already a meeting of minds, the contract is perfected.
d. The contract is unenforceable.

4. G was appointed guardian of S, the latter being 16 years old, S sold his parcel of land in writing
to B valued at P100,000 for P65,000. What is the status of the contract?
a. Rescissible
b. Void
c. Unenforceable
d. Enforceable
e. Voidable
i. One of the parties who entered into the contract was incapable of giving
consent.

5. Which of the following obligations of the vendor cannot be waived?


a. To allow the buyer to examine the goods sold
b. To pay the expenses of the deed of sale
c. To warrant the thing sold
d. To transfer ownership to the buyer

6. When delivery takes place by the mere consent of agreement of contracting parties as when the
vendor merely points to the thing sold which shall thereafter be a the control and disposal of
the vendee if the thing sold cannot be transferred to the possession of the vendee at the time of
the sale, delivery is effected:
a. By actual delivery
i. Delivery or transfer of a thing from hand to hand if it is movable, or by certain
material and possessory acts of the grantee performed in the presence and
with the consent of the grantor if it is movable.
ii. “real tradition”
b. By traditio longa manu
i. The grantor pointing out to the grantee the thing which is delivered which at
the time must be within sight.
c. By traditio brevi manu
i. Delivery of movable property takes place when the vendee had the thing
already in his possession before the sale took place, not as owner but as
lessee, borrower or as depositary.
d. Constitutum possesorium

Practice Questions and Suggested Answers 48


CIVIL LAW REVIEW II

i. Vendor remains in possession of the property sold by virtue of a lease contract


agreement with the vendee.

7. Lucy is a lessee of a store in a shopping mall. She only sells/assigns her rights to Salome. She
then allows Salome to occupy the stall. The delivery here is called:
a. Quasi-tradition
i. This is used to indicate the exercise of a right by the grantee with the
acquiescence of the grantor.
b. Actual delivery
i. Delivery or transfer of a thing from hand to hand if it is movable, or by certain
material and possessory acts of the grantee performed in the presence and
with the consent of the grantor if it is movable.
ii. “real tradition”
c. Traditio longa manu
i. The grantor pointing out to the grantee the thing which is delivered which at
the time must be within sight.
d. Traditio brevi manu
i. Grantee is already in possession of the thing under a title which is not of
ownership, such as when the lessee purchases from the lessor the object of
the lease.

8. Ownership of the thing sold is transferred/acquired/retained:


a. Retained by the seller in “sale or return”
i. Wrong. In a sale or return, ownership passes to the buyer upon delivery, but
the buyer can revest the title to the seller.
b. Transferred to the buyer upon constructive or actual delivery of the thing sold
c. Acquired by the buyer upon perfection of the contract
i. Wrong. It is fundamentally wrong. There was no delivery, but only mere
perfection of the contract. As a general rule, ownership is transferred upon
delivery.
d. Transferred to the buyer upon acceptance of the price
i. Wrong. It is fundamentally wrong. There was no delivery, but only mere
perfection of the contract. As a general rule, ownership is transferred upon
delivery.

9. Which of the following statements is correct:


a. In a contract of sale, full payment of the price is in the nature of suspensive condition in
that the seller is obligated to transfer ownership of the thing sold upon payment.
i. Wrong. This only applies in an absolute sale. In a conditional sale, full payment
of the price may not be the only condition to be fulfilled.
b. The seller need not be the owner of the thing sold at the perfection of the sale.
i. This is not absolutely correct. There are different types of perfection which
requires immediate delivery.
ii. What the law requires of the seller is the right to transfer ownership, not
necessarily ownership of the thing sold.
c. There may be a transfer of ownership over the thing even if the seller has not
delivered the thing sold to the buyer.

Practice Questions and Suggested Answers 49


CIVIL LAW REVIEW II

d. In a contract of sale, the buyer becomes the owner of the thing sold upon full payment
of the purchase price.
i. Wrong because this does not cover conditional sales.

10. Which of the following statements is not correct:


a. Actual delivery of the thing or payment of the price is not required for the perfection of
the sale.
i. The contract of sale is perfected at the moment there is a meeting of the
minds upon the thing which is the object of the contract and upon the price.
(Art. 1475)
b. A stipulation that even when the object is delivered to the buyer, ownership will not
pass until the price is fully paid is not valid.
c. A sale is consummated upon delivery of the thing and the payment of the purchase
price.
i. In a conditional sale, payment of the price and/or delivery of the thing sold
may not be the only conditions to be fulfilled for the consummation of the
sale.
d. Sales through letters or telegrams are deemed perfected only when acceptance by the
buyer is known to the seller.

11. The delivery of movable property may be made by the mere consent or agreement of the
contracting parties, if the thing sold cannot be transferred to the possession of the vendee at
the time of the sale. This mode of delivery is known as:
a. Actual delivery
b. Traditio constitutum possesorium
i. Vendor remains in possession of the property sold by virtue of a lease contract
agreement with the vendee.
c. Traditio longa manu
d. Traditio brevi manu
i. Grantee is already in possession of the thing under a title which is not of
ownership, such as when the lessee purchases from the lessor the object of
the lease.
e. Formal delivery

12. In a sale of an incorporeal property, the following are the modes of delivery, except:
a. Execution of public instrument
b. Placing of the titles of ownership in the possession of the vendee
c. Use by the vendee of his rights, with the vendor’s consent
d. Actual delivery
e. None of the above

13. In this kind of sale, the ownership passes to the buyer on delivery, but he may revest the
ownership in the seller by returning or tendering the goods within the time fixed in the contract,
or, if no time has been fixed, within a reasonable time.
a. Conditional sale
i. Conditions are imposed by the seller before ownership will pass. Normally, the
condition is the full payment of the price.

Practice Questions and Suggested Answers 50


CIVIL LAW REVIEW II

ii. Ownership automatically passes to the buyer from the moment the condition
happens. There is no need for another contract to be entered into.
b. Sale or return
c. Sale on trial
i. Ownership passes to the buyer when (1) he signifies his acceptance or does
any other act adopting the transaction; (2) retains the goods without giving
notice of rejection, then if a time has been fixed for the return of the goods,
on the expiration of such time, and if no time has been fixed, on the expiration
of a reasonable time. (Art. 1502)
d. Contract to sell
i. It is considered a special kind of conditional sale, it is a peculiar kind of sale
because despite the happening of the condition and actual delivery, the buyer
does not automatically acquire ownership. If condition happens, the right of
the buyer is to compel the seller to execute a final deed of sale. So ownership
does not automatically pass.
e. Absolute sale
i. Seller does not reserve his title over the thing sold and thus, upon delivery of
the thing, ownership passes regardless of whether or not the buyer has paid.

14. For ownership to pass, the seller must have the right to sell:
a. Before the perfection of the sale
b. At the time of perfection
c. Upon payment
d. At the time ownership is to pass
e. Upon registration of the sale

15. When a person who is not the owner of a thing sells or alienates and delivers it, and later the
seller or grantor acquires title thereto, such title passes by operation of law to the buyer or
grantee. This is:
a. Estoppel in pais
i. By the principle of estoppel, a person is precluded from denying that another
person has authority to sell because of his acts. Also known as “Estoppel in
Pais” which is a kind of equitable estoppel because of the acts /
representation of the owner, he may not later on deny the authority of the
3rd person.
b. Estoppel by deed
i. When the seller who was not the owner at the time of the sale, acquires
ownership, automatically, ownership passes to the buyer by operation of law.
However, Article 1434 requires delivery to the buyer.
c. Estoppel by record
i. Case where the seller was estopped from denying the authority to sell due to
his prior testimony made in court.
d. Equitable estoppel
e. None of the above

16. In a CIF arrangement, delivery is considered to take place at:


a. Place of perfection of the sale
b. Place of payment

Practice Questions and Suggested Answers 51


CIVIL LAW REVIEW II

c. Port of origin
d. Port of destination
i. This applies in an FOB arrangement.
e. None of the above
17. In this mode of delivery, the seller would remain in possession of the thing after the sale:
a. Actual delivery
b. Traditio constitutum possesorium
c. Traditio longa manu
d. Traditio brevi manu
e. Formal delivery

18. This seller has a statutory power to sell:


a. Sheriff
i. This seller has the authority given by the court.
b. Agent
i. This seller was given the authority to sell by the owner.
c. Auctioneer
d. Notary public
i. He may be the owner but he may have the authority of the law to sell, known
as “Statutory Power to Sell” (Article 1505). Examples: Notary public in pledge,
liquidators, guardians and receivers.

19. Ownership passes upon delivery in:


a. Conditional sale
i. Conditions are imposed by the seller before ownership will pass.
b. Sale on trial
i. Ownership passes to the buyer when (1) he signifies his acceptance or does
any other act adopting the transaction; (2) retains the goods without giving
notice of rejection, then if a time has been fixed for the return of the goods,
on the expiration of such time, and if no time has been fixed, on the expiration
of a reasonable time. (Art. 1502)
c. Absolute sale
i. Seller does not reserve his title over the thing sold and thus, upon delivery of
the thing, ownership passes regardless of whether or not the buyer has paid.
d. None of the above

20. Who can transfer ownership by virtue of a sale?


a. Seller
b. Buyer
i. Consider “sale or return”
c. One with a right to sell
d. All of the above
e. None of the above

21. The following are requisites for a valid exercise of the right of stoppage in transitu, except:
a. seller is unpaid
b. goods are in transit
c. seller has not parted with the possession of the goods

Practice Questions and Suggested Answers 52


CIVIL LAW REVIEW II

d. buyer is insolvent

22. One of the following is not a remedy granted to an unpaid seller:


a. right of stoppage of goods in transit
b. right of lien over the goods
c. right of resale
d. right to demand a security for the payment of the price

Under the Civil Code, the following are the remedies of an unpaid seller: (1) retain
the thing in his possession; (2) stoppage in transitu; (3) resale; and (4) rescission.
There is no mention of a right to demand security for the payment of the price.

23. One of the following is not an alternative remedy available to a seller of personal property sold
on instalment if the buyer defaults.
a. to go to court and ask the court to order delivery of the property
b. to foreclose the chattel mortgage
c. to sue for specific performance
d. to take possession of the property and forfeit the amount paid by buyer

24. The following are the alternative remedies, except one, available to the buyer in breach of
warranty by the seller.
a. keep the goods and ask for damages
b. refuse to accept the goods and ask for damages
c. rescind the sale and retain the goods

The remedy provided by law is to rescind the sale and refuse to receive the
goods, or if the goods have already been received, return them or offer to
return them to the seller and recover the price or any part thereof which has
been paid. (Art. 1599)

d. keep the goods and set up against the seller by way of recoupment in price

25. If redemption is made, which of the following will not be paid by the seller to the buyer?
a. price paid by the buyer
b. expenses in the execution of the sales contract paid by the buyer
c. all necessary expenses on the thing sold and to be redeemed
d. interest of the price paid by the buyer

The law does not require the redemptioner to pay for any interest.

26. M, N and O are co-owners of a parcel of land proindiviso. M sold his 1/3 share in the land to N in
an absolute sale. Which is correct?
a. The deed of sale between M and N is void because it was not made in favor of a third
party.
b. O may exercise his right of redemption on the interest sold by M to N.
c. O may redeem only ½ of the interest sold by M to N.
d. O cannot exercise the right of redemption because the sale was made in favor of a co-
owner.

Practice Questions and Suggested Answers 53


CIVIL LAW REVIEW II

27. An unpaid seller has the following rights, except:


a. a lien on the goods after he has parted with the possession of them

Unpaid seller cannot have a lien on the goods if he does not have possession
over them.

b. in case of insolvency of the buyer, a right of stopping the goods in transitu


c. a right to rescind, under the circumstances provided by law
d. a right of resale, under the circumstances provided by law

28. The unpaid seller is not entitled to retain the possession of the goods where:
a. the goods have been sold without stipulation as to credit
b. the goods have been sold on credit, but the term of credit has expired
c. the buyer is insolvent
d. the possession of the seller on the goods is only as agent or bailee for the buyer
e. none of the above

Art. 1527.

29. What mode of extinguishing a contract of sale is effected when a person is subrogated upon the
same terms and conditions stipulated in the contract in the place of one who acquires a thing by
onerous title?
a. compensation
b. conventional redemption
c. novation
d. legal redemption
e. none of the above

30. A sold to B a specific car for P 250,000 in 10 equal monthly installments. B failed to pay the 3 rd
installment. The right of A is:
a. cancel the sale
b. demand payment from B
c. (missing)
d. (missing)

31. In a contract of sale, a warranty against eviction is:


a. an essential element
b. a natural element
c. an accidental element
d. a formal requisite

32. It is an affirmation of fact or promise by the seller relating to the thing which has a natural
tendency to induce the buyer to purchase the same, relying on such promise or affirmation.
a. condition
b. false representation
c. express warranty
d. seller’s talk

Practice Questions and Suggested Answers 54


CIVIL LAW REVIEW II

33. In a contract of sale of a specific carabao, the seller was obliged to deliver the animal on
December 31, 2003. In November 2003, the carabao had an offspring. The offspring belongs to:
a. Seller, because the sale is already perfected prior to its truth.
b. Seller, because the fruit arose before the obligation to deliver arises.
c. Buyer, because the fruit arose after the perfection of the contract.
d. Buyer, if he pays the purchase price.

Under Art. 1164, the creditor has a right to the fruits of the thing from the
time the obligation to deliver it arises. However, he shall acquire no real right
over it until the same has been delivered to him.

34. There is no warrant in:


a. “as is, where is sale”
a. Sale which means as it is found, where it is found
b. sale of second hand articles
c. animals sold at fairs
d. sale by virtue of authority in fact or law
e. none of the above

35. Persons not liable for breach of warranty, except:


a. Sheriff
b. Auctioneer
c. Mortgagee
d. Pledgor
e. none of the above

36. When does the buyer of a thing has the right to the fruits of the thing bought? From the time
the obligation to deliver the thing arises.

37. If an immovable property is sold to two persons, ownership shall belong to the person ____.
Who first recorded the sale and bought the property in god faith.

38. A contract of sale is perfected ____.

As a general rule, when there is meeting of the minds between the vendor and the
vendee on the object of the sale and the price certain.

39. It is a contract by virtue of the terms which the parties thereto promise and obligate themselves
to enter into another contract at a future time, upon the happening of certain events, or the
fulfillment of certain conditions.

Contract of option or option-contract.

40. A contract of sale is not necessarily:


a. Consensual
b. Bilateral
c. Commutative

Practice Questions and Suggested Answers 55


CIVIL LAW REVIEW II

d. Onerous
e. None

41. When the goods are delivered to the buyer, the ownership passes to the buyer in ___.

Sale or return.

42. The following may not be valid objects of a contract of sale, except:
a. objects outside the commerce of men
b. illicit things
c. Future goods.
d. impossible service

43. …

44. S makes an offer to B on January 1, 1996. B makes known his known his acceptance in a letter
sent on January and received by S on January 10. Meantime on January 5, S became insane.

There was no perfected contract of sale since there was no meeting of the minds
between the party, applying the cognition theory where there is a perfected contract
of sale if the acceptance of the buyer is made known to the seller. Unlike
manifestation theory where acceptance by the buyer perfects the contract already.

45. Which of the following obligations of the vendor cannot be waived.

to deliver the thing to the buyer

Transfer of ownership of the object of the sale is necessary in the performance of the
obligation of the vendor and transfer of ownership is effected only through delivery.
There is always delivery in a contract of sale it is either actual or constructive.

46. Ownership of the thing sold is transferred/acquired/retained:


a. retained by seller in sale or return
b. transferred to buyer upon constructive or actual delivery
c. acquired by the buyer upon perfection of contract
d. transferred to buyer upon acceptance of price

47. Which of the following statements is correct?


a. in COS, the full payment of the price is in the nature of suspensive
b. seller need not be the owner of the thing sold at the perfection of the sale
c. there may be a transfer of ownership over the thing even if the seller has not delivered
the thing sold to the buyer
d. in COS, the buyer becomes the owner of the thing sold upon full payment of the
purchase price

48. In a contract of sale of a specific carabao, the seller was obliged to deliver the animal on Dec 31
2003. In November 2003, the carabao had an offspring. The offspring belongs to ___.

Practice Questions and Suggested Answers 56


CIVIL LAW REVIEW II

Seller, because the fruit arouse before the obligation to deliver arises.

49. Sale is distinguished from dation in payment in that in dation in payment ___.

There is a pre-existing obligation or credit.

50. There is a contract of sale ___.


a. When goods are delivered to distributor on consignment
b. When the manufacturer delivers the foods to an agent where the former retains the
ownership and dictates the term of the sale
c. When foods are delivered to a person on charge account
d. When goods are delivered to an agent to be sold by him and the agent is not liable to
the manufacturer of the goods

51. The following persons cannot acquire by purchase, even at a public or judicial auction, either in
person or through the mediation of another, except ___

Agents, the property whose administration or sale may have been entrusted to them.

52. Araneta wrote a letter to Bascon wherein he offered to sell a piece of land. In Araneta’s letter he
gives Bascon a period of 2 months within which to pay the price of 500k. After 50 days, Araneta
told Bascon that he is increasing the price of the land to 700k. Can Bascon compel Araneta to
accept the 500k first offered by Araneta and execute the deed of sale?

No, Bascon did not signify his acceptance of offer.

53. A characteristic of COS which involves exchange of value, it is ___.

Commutative

54. A case where COS may not necessarily be in writing to be enforceable.


a. Sale of 100 piculs of sugar at 400 per picul when there is partial delivery; even if there
is no partial delivery, the amount is already more than 500 and covered by the statue
of frauds.
b. Sale whereby its term cannot be performed within one year from the making thereof
c. Sale of land
d. All of the Above
e. None of the Above

55. In a contract of sale, the nature of the object of the sale will not be relevant in
a. Maceda Law
b. Recto Law
c. Double Sales
d. Statute of Frauds
e. None of the Above

Practice Questions and Suggested Answers 57


CIVIL LAW REVIEW II

STATUS OF CONTRACTS

1. The guardian purchases the property of his ward.


a. VOID.

2. The purchaser of a private agricultural land is a former Filipino citizen.


a. VALID.

3. Contracts of sale with a false cause.


a. VOID.

4. Sale of a car where the price is in Chinese Yuan.


a. VALID.

5. Oral sale of an immovable for P450.


a. UNENFORCEABLE.

6. Sale of large cattle in a private instrument.


a. VOID.

7. The guardian sold the property of his ward valued at P100,000 to another person for the price of
P70,000.
a. UNENFORCEABLE.

8. A deed of absolute sale which does not contain a provision as to the price.
a. VALID.

9. Sale of hereditary rights of an heir to the executor of the will of the decedent.
a. VOID.

10. Sale of a property in litigation to the court stenographer of the court where the case is pending.
a. VOID.

Practice Questions and Suggested Answers 58


CIVIL LAW REVIEW II

SALES II

MODIFIED TRUE OR FALSE

A. Both are true


B. Both are false
C. No. 1 is true, No. 2 is false
D. No. 1 is false, No. 2 is true

1. Statements:
 No affirmation of the value of the thing, nor any statement purporting to be a statement of the
seller’s opinion only, shall be construed as a warranty.
o FALSE, it will be considered a warranty when the seller made such affirmation or
statement as an expert and it was relied upon by the buyer. Art. 1546.

 The sale of animals suffering from contagious diseases shall be void.


o TRUE, Art. 1575.

2. Statements:
 For the vendor to be held liable for breach of warranty against eviction, the judgment must be
based on a right prior to the sale.
o FALSE, judgment may also be based on an act imputable to the vendor.

 The vendee need not appeal from the decision in order that the vendor may become liable for
eviction
o TRUE, Art. 1549.

3. Statements:
 When adverse possession had been commenced before the sale but the prescriptive period is
completed after the transfer, the vendor shall not be liable for eviction.
o TRUE, Art. 1550.

 There are no implied warranties in sale of animals at fairs or at public auctions, or of live stock
sold as condemned.
o FALSE, only warranties against hidden defects are not provided for such sales.

4. Statements:
 If the property is sold for nonpayment of taxes due, the vendor is liable for eviction.
o FALSE, such nonpayment of taxes due shall not be made known to the vendee before
the sale.

 If two or more animals are sold together, whether for a lump sump or for a separate price for
each of them, the redhibitory defect of one shall only give rise to its redhibition and not that of
the others.
o FALSE, the statement will only be true if it should NOT appear that the vendee would
not have purchased the sound animal or animals without the defective one. This will

Practice Questions and Suggested Answers 59


CIVIL LAW REVIEW II

be presumed when a team, yoke, pair, or set is bought, even if a separate price has
been fixed for each one of the animals composing the same. Art. 1572.

5. Statements:
 Any stipulation exempting the vendor from the obligation to answer for eviction shall be void.
o FALSE, vendor must act in bad faith to render such stipulation void. Art. 1553.

 Should the vendee have made the waiver with knowledge of the risks of eviction and assumed
its consequences, the vendor shall not be liable.
o TRUE, Art. 1554.

Practice Questions and Suggested Answers 60


CIVIL LAW REVIEW II

MULTIPLE CHOICE QUESTIONS

6. The following cannot be held liable for breach of warranty except:


a. sheriff
b. auctioneer
c. mortgagor ??
d. pledgee
e. none of the above

The law would specifically exempt certain persons from liability for breach of warranty like
sheriff, auctioneer, mortgagee, pledgee and other persons who sell by virtues of an authority
of law like notary public because they are not really selling for themselves, they are selling on
behalf of another person.

7. The redhibitory action, based on the faults or defects of animals must brought within:
a. forty days from the date of their delivery to the vendee. (Art. 1577)
b. forty days from the date of the contract
c. six months from the date of their delivery to the vendee.
d. six months from the date of the contract
e. one year from the date of the contract

8. If there is a stipulation exempting the vendor from the obligation to answer for eviction and the
vendor acted in bad faith the vendor shall be liable for the following:
a. Value of the thing at the same time of eviction
b. Income or fruits, if he has been ordered to deliver them to the party who won the suit against
him
c. Expenses of the contract, if the vendee has paid them, and
d. Damages and interest, and ornamental expenses, if the sale was made in bad faith
e. All of the above. (Art. 1555)

9. There is no implied warranty as to hidden defects/quality in the following, except:


a. “as is, where is” sales
b. sale of second hand articles
c. sale by virtue of authority in fact or law (Art. 1570)
d. animals sold at fairs or at public auctions (Art. 1574)
e. when validly suppressed by parties

10. The buyer is deemed to have accepted the goods


a. when he intimates to the seller that he is accepting them.
b. when he does any act in relation to the goods which is inconsistent with the ownership of the
seller
c. when he retains the goods after the lapse of a reasonable time without intimating to the
seller that he has rejected them.
d. all of the above. (Art. 1585)
e. none of the above.

Practice Questions and Suggested Answers 61


CIVIL LAW REVIEW II

MACEDA LAW

TRUE OR FALSE

1. If the place of payment should not have been stipulated, the payment must be made at the
place of the delivery of the thing sold.
2. The buyer of goods is not bound to accept delivery by installments.
3. Where the goods are delivered to the buyer, he is not deemed to have accepted them unless
and until he has had a reasonable opportunity of examining them for the purpose of
ascertaining whether they are in conformity with the contract.
4. Where there is a breach of warranty by the seller, the buyer may, refuse to accept the goods,
and maintain an action against the seller for damages for the breach of warranty.
5. Acceptance of the goods by the buyer shall not discharge the seller from liability in damages or
other legal remedy for breach of any promise or warranty in the contract of sale.
6. If, after acceptance of the goods, the buyer fails to give notice to the seller of the breach in any
promise of warranty within a reasonable time after the buyer knows, or ought to know of such
breach, the seller shall not be liable therefor.
7. The Maceda Law can only be invoked if the buyer had paid at least two years of installments
before default.
8. The right to a grace period shall be exercised by the buyer, under the Maceda Law, only once in
every five years of the life of the contract, and its extensions, if any.
9. Down payments, deposits or options in a sale covered by the Maceda Law, shall be included in
the computation of the total number of installment payments made.
10. A sale of realty on credit will be covered by the Maceda Law.

MULTIPLE CHOICE QUESTIONS

11. The buyer is deemed to have accepted the goods when:


a. he intimates to the seller that he has accepted them
b. when the goods have been delivered to him, and he does any act in relation to them which is
inconsistent with the ownership of the seller.
c. when, after the lapse of a reasonable time, he retains the goods without intimating to the
seller that he has rejected them.
d. all of the above
e. none of the above

12. The vendee shall owe interest for the period between the delivery of the thing and the payment
of the price, in the following cases, except:
a. Should it have been so stipulated
b. Should the thing sold and delivered produce fruits or income
c. From the time the obligation to pay the price becomes due and demandable
d. None of the above

13. The Realty Installment Buyer Act may apply to:


a. a sale of industrial lots
b. a sale of commercial buildings
c. sales to tenants
d. all of the above

Practice Questions and Suggested Answers 62


CIVIL LAW REVIEW II

e. none of the above

14. Under the Maceda Law, the buyer is entitled to a grace period of not less than:
a. 30 days
b. 60 days
c. 90 days.
d. one year
e. two years

15. The cash surrender value is a minimum of:


a. 20% of the total payments made
b. 30% of the total payments made
c. 40% of the total payments made
d. 50% of the total payments made
e. 92% of the total payments made

Practice Questions and Suggested Answers 63


CIVIL LAW REVIEW II

STATUS OF CONTRACTS OF SALE

1. There may be a contract of sale of goods, whose acquisition by the seller depends upon a
contingency which may or may not happen.
a. TRUE. Express provision 1462(2).

2. The sale of expectancy is voidable.


a. FALSE. The sale of vain hope or expectancy is void. (1461(2))

3. Things having a potential existence may be the object of the contract of sale.
a. TRUE. Express provision 1461(1).

4. A contract of sale is void when the object is neither particularly designated nor physically
segregated from all others of the same class.
a. FALSE. The requisite that a thing be determinate is satisfied if at the time the contract
is entered into, the thing is capable of being made determinate without the necessity
of a new or further agreement between the parties. (1460(2))

5. If the price is simulated, the sale is void, but the act may be shown to have been in reality a
donation, or some other act or contract.
a. TRUE. Express provision 1471.
i. Alternative answer: FALSE. Parties may be held bound by the false price
indicated in the instrument, especially when the interest of the Government
or third persons would be adversely affected by the reformation of the
contract. (Spouses Doromal v. CA)

6. The husband and the wife cannot sell property to each other.
a. FALSE. The following are exceptions to the rule – a
i. Separation of property in the marriage settlement; and
ii. Judicial separation of property (Art. 1490)

7. A contract of sale entered into by a guardian over the property of his ward is rescissible if the
price is only 70% of the value of the property.
a. TRUE.

8. A contract of sale is void when the executor is the vendee and the object of the sale is a
property of the estate under his administration.
a. TRUE. Relative Incapacity. (Art. 1491, par. 3)

9. A contract of sale if void when the vendee is not a Filipino citizen and the object of the sale is a
private land.
a. FALSE. An exception is when a formerly natural-born Filipino citizen who lost his
Filipino citizenship. (Estate of Serra v. Heirs of Hernaez)
i. Alternative answer: TRUE. The question depends on how you appreciate “not
a Filipino citizen”.

Practice Questions and Suggested Answers 64


CIVIL LAW REVIEW II

10. If the fixing of the price is left to the discretion of one of the parties, the contract will not be
perfected.
a. FALSE. The fixing of the price can never be left to the discretion of one of the
contracting parties. However, if the price fixed by one of the parties is accepted by the
other, the sale is perfected. (1473)

Practice Questions and Suggested Answers 65


CIVIL LAW REVIEW II

REMEDIES

TRUE OR FALSE

16. It is not essential to the validity of a resale that notice of the time and place of such resale
should be given by the seller to the original buyer.
a. TRUE.

17. It is not essential to the validity of resale that notice of an intention to resell the goods be given
by the seller to the original buyer.
a. TRUE.

18. The unpaid seller cannot directly or indirectly buy the goods in the exercise of the right to resell.
a. TRUE.

19. The seller of goods is deemed to be an unpaid seller when a bill of exchange or other negotiable
instrument has been received as conditional payment.
a. FALSE.

20. The unpaid seller may exercise his right of stoppage in transitu by obtaining actual possession of
the goods.
a. TRUE.

21. The unpaid seller shall not be liable to the original buyer for any profit made by the resale of the
goods.
a. TRUE.

22. A lien on the goods of the unpaid seller of goods can be exercised even if he is in possession of
them merely as a depository.
a. TRUE.

23. The remedies of the unpaid seller of goods can no longer be invoked if ownership in the goods
have passed already to the buyer.
a. FALSE.

24. The seller of goods is deemed to be an unpaid seller when the whole of the price has not been
paid.
a. TRUE.

25. The remedies of an unpaid seller are alternative in character.


a. FALSE.

MULTIPLE CHOICE QUESTIONS

26. That the buyer is insolvent is a requirement for the exercise of the:

Practice Questions and Suggested Answers 66


CIVIL LAW REVIEW II

a. A right to retain the goods


b. A right of stopping the goods in transitu
c. A right of resale
d. A right to rescind the sale
e. A right to demand for the payment of the unpaid price

27. The unpaid seller of goods loses his lien , except:


a. When he delivers the goods to a carrier or other bailee for the purpose of transmission
to the buyer without reserving the ownership in the goods or the right to the possession
thereof
b. When the buyer or his agent lawfully obtains possession of the goods.
c. When he has obtained judgment or decree for the price of the goods.
d. By waiver thereof
e. None of the Above

28. Goods are no longer in transit:


a. If the buyer, or his agent in that behalf, obtains delivery of the goods before their arrival
at the appointed destination
b. If, after the arrival of the goods at the appointed destination the carrier or other bailee
acknowledges to the buyer or his agent that he holds the goods on his behalf and
continues in possession of them as bailee for the buyer or his agent
c. If the carrier or other bailee wrongfully refuses to deliver the goods to the buyer or his
agent in that behalf
d. All of the above
e. None of the Above

29. An unpaid seller cannot exercise the right to resell if:


a. The seller has parted possession of the goods
b. The goods are of perishable nature
c. The seller expressly reserves the right of resale in case the buyer should make default
d. The buyer has been in default in the payment of the price for an unreasonable time
e. All of the above
f. None of the above

30. The unpaid seller’s right of lien or stoppage in transitu is not affected by any sale, or other
disposition of the goods which the buyer may have made, unless:
a. The seller has parted possession of the goods
b. A negotiable document of title has been issued for goods, which were sold to a
purchaser for value in good faith to whom such document has been negotiated
c. The goods are of perishable nature
d. The seller expressly reserves the right of resale in case the buyer should make default
e. The buyer has been in default in the payment of the price for an unreasonable fare

Practice Questions and Suggested Answers 67


CIVIL LAW REVIEW II

EXTINGUISHMENT OF CONTRACT OF SALE

TRUE OR FALSE

1. Legal redemption shall take place when the vendor reserves the right to repurchase the thing
sold.
a. FALSE.

2. The vendor who repurchases the thing shall respect the leases constituted by the vendee.
a. FALSE.

3. Legal redemption is the right to be subrogated, upon the same terms and conditions stipulated
in the contract, in the place of one who acquires a thing by purchase only.
a. FALSE.

4. Should there have been no fruits at the time of the sale and some exist at the time of
redemption, they shall be prorated between the redemptioner and the vendee.
a. TRUE.

5. The contract shall be presumed to be a sale with a right to repurchase when the price in a
mortgage is unusually inadequate.
a. FALSE.

6. The vendor cannot avail himself of the right of repurchase without returning to the vendee the
price of the sale and the necessary and useful expenses made on the thing sold.
a. TRUE.

7. In case of doubt, a contract purporting to be a sale with right to repurchase shall be construed
as an equitable mortgage.
a. TRUE.

8. The vendee is subrogated to the vendor’s rights and actions.


a. TRUE.

9. The creditors of the vendor cannot make use of the right of redemption against the vendee,
until after they have exhausted the property of the vendor.
a. TRUE.

10. The right of redemption of co-owners excludes that of adjoining owners.


a. TRUE.

MULTIPLE CHOICE QUESTIONS

11. If the transaction is an equitable mortgage, the remedy is:


a. declaration of nullity

Practice Questions and Suggested Answers 68


CIVIL LAW REVIEW II

b. annulment
c. rescission
d. any of the above
e. none of the above

12. In conventional redemption, the period for redemption, in the absence of an express agreement
shall last:
a. one year from the date of the contract
b. four years from the date of the contract
c. one year from the registration of the contract
d. four years from the registration of the contract
e. none of the above

13. Should there be an agreement as to the period of redemption, the period cannot exceed:
a. one year
b. four years
c. ten years
d. thirty years
e. none of the above

14. In a sale of a parcel of land with a right to repurchase, there being no stipulation to the contrary,
ownership passes to the vendee upon:
a. the perfection of the sale
b. upon delivery
c. upon payment of the price
d. upon the lapse of the period to repurchase
e. upon the order of the court

15. The remedy of the creditors of the vendor as to the right of redemption against the vendee is:
a. a cumulative remedy
b. a principal remedy
c. a subsidiary remedy
d. all of the above
e. none of the above

Practice Questions and Suggested Answers 69


CIVIL LAW REVIEW II

LEASE

TRUE OR FALSE

1. A contract where one of the parties binds himself to render to the other some service is a
contract for a piece of work.
a. FALSE, the statement pertains to a contract of lease of service.
i. In a contract for a piece of work, the contractor binds himself to execute a
piece of work for the employer, in consideration of a certain price or
compensation. The contractor may either employ only his labor or skill, or also
furnish the material. (Art. 1713)
ii. In the lease of work or service, one of the parties binds himself to execute a
piece of work or to render to the other some service for a price certain. (Art.
1644)

2. No lease for more than ninety-nine years shall be valid.


a. TRUE, Art. 1643.

3. Fungible goods cannot be the subject matter of a contract of lease.


a. TRUE, Art. 1665. --- according to source.
i. Alternative answer: FALSE, Art. 1645. When the lease of the thing, although
fungible, is only for exhibit or it is an accessory to an industrial establishment,
it can be the subject matter of a lease.

4. The guardian cannot lease the property of his ward for himself.
a. FALSE, although Art. 1646, in relation to Art. 1491, provides that a guardian is
disqualified to become a lessee of a property of his ward, Art. 1647 will allow the
guardian to lease such property if with proper court authority.

5. If a lease is to be recorded in the Registry of Property, the father or guardian, as to the property
of the minor or ward, cannot constitute the same without proper authority.
a. TRUE, Art. 1647.

6. When in the contract of lease of things there is no express prohibition, the lessee may assign the
lease, in whole or in part, without prejudice to his responsibility for the performance of the
contract toward the lessor.
a. FALSE, Art. 1649 and 1650.
b. The lessee cannot assign the lease without the consent of the lessor, unless there is a
stipulation to the contrary. (Art. 1649)
c. When in the contract of lease of things there is no prohibition, the lessee may sublet
the thing leased. (Art. 1650)
d. As a general rule:

Practice Questions and Suggested Answers 70


CIVIL LAW REVIEW II

i. Lessee CANNOT assign the lease.


ii. Lessee CAN sublease.

7. The sublessee shall not be responsible beyond the amount of rent due from him, in accordance
with the terms of the sublease, at the time of the extra-judicial demand by the lessor.
a. TRUE, Art. 1652. --- according to source.
i. Alternative answer: FALSE, Art. 1652, par. 2.

8. The provisions governing warranty, contained in the title on Sales, shall be applicable to the
contract of lease.
a. TRUE, Art. 1653.

9. The lessor is obliged to make all the necessary repairs in order to keep the thing suitable for the
use to which it has been devoted.
a. FALSE, parties’ stipulation. (Art. 1654, par. 2)
b. cockpit case? --- according to source.

10. If the thing leased is destroyed partially by fortuitous event, the lessee may choose between a
proportional reduction of the rent and specific performance.
a. FALSE, the remedies of the lessee are only proportional reduction of the rent OR
rescission, not specific performance. (Art. 1655)

11. There is a mere act of trespass when the third person claims no right whatever.
a. TRUE, Art. 1664, par. 2.

12. The lessee has the burden of proving that the deterioration or loss of the thing leased was
without his fault.
a. FALSE, the presumption does not apply when happens during calamity.
i. This burden of proof on the lessee does not apply when the destruction is due
to earthquake, flood, storm or other natural calamity. (Art. 1667)

13. If at the end of the contract the lessee should continue enjoying the thing leased for fifteen days
with the acquiescence of the lessor, and unless a notice to the contrary by either party has
previously been given, it is understood that there is an implied new lease, for the period of the
original contract.
a. FALSE, the implied new lease is not for the period of the original contract. It will
depend if the land is rural or urban –
i. If rural land, see Art. 1682.
ii. If urban land, see Art. 1687.

Practice Questions and Suggested Answers 71


CIVIL LAW REVIEW II

14. The purchaser of a piece of land which is under a lease that is not recorded in the Registry of
Property may terminate the lease, save when there is a stipulation to the contrary in the
contract of sale.
a. FALSE, lease may be terminated in such case when there is a stipulation to the
contrary in the contract of sale OR when the purchaser knows of the existence of the
lease. (Art. 1676)

15. With regard to ornamental expenses, the lessee shall not be entitled to any reimbursement, but
he may remove the ornamental object, provided no damage is caused to the principal thing, and
the lessor does not choose to retain them by paying their value at the time the lease is
extinguished.
a. TRUE, Art. 1678.

16. The lessee shall have the right to reduction of the rent by reason of the loss of more than one-
half of the fruits through extraordinary and unforeseen fortuitous event, save always when
there is a specific stipulation to the contrary.
a. TRUE, Art. 1680.

17. If the period for the lease of a rural land has been fixed, it is understood to be from year to year,
if the rent agreed upon is annual, from month to month, if it is monthly from week to week, if
the rent is weekly, and from day to day, if the rent is to paid daily.
a. FALSE, statement pertains to urban land, not rural land. (Art. 1687)
i. Also, the provision will only apply if the period has NOT been fixed.

18. An agreement waiving or limiting the contractor’s liability for any defect in the work is void.
a. FALSE, statement will apply when the contractor acted fraudulently. Art. 1716.

19. The contractor is responsible for the damages if the edifice falls, within fifteen years from the
completion of the structure, on account of defects in the construction or the use of materials of
inferior quality furnished by him, or due to any violation of the terms of the contract. If the
engineer or architect supervises the construction, he shall be jointly liable with the contractor.
a. FALSE, the supervising engineer or architect shall be held solidarily liable with the
contractor. (Art. 1723)

20. He who has executed work upon a movable has a right to retain it by way of pledge until he is
paid.
a. TRUE, Art. 1731.

Practice Questions and Suggested Answers 72


CIVIL LAW REVIEW II

CREDIT TRANSACTIONS: GENERAL PRINCIPLES

MODIFIED TRUE OR FALSE.

1. D
 A contract of loan is purely personal in character. – False (1939)
 An accepted promise to deliver something by way of simple loan is binding upon parties, but
commodatum shall not be perfected until the delivery of the object of the contract. – True 1934

2. B
 When there are two or more bailees to whom a thing is loaned in the same contract, they are
liable jointly. – False (1945) Solidarily
 The bailor cannot demand the return of the thing loaned till after the expiration of the period
stipulated. – False (1946, 1948) After expiration of period stipulated
After accomplishment of the use for which the commodatum has been constituted
Urgent need
3. D Acts of ingratitude by bailee

 The bailor who, knowing the flaws of the thing loaned, shall be liable to the latter for the
damages which he may suffer by reason thereof. – False (1951) Only if the bailor does not advise the bailee
 In the determination of the interest, if it is payable in kind, its value shall be appraised at the
current price of the products or goods at the time and place of payment. – True (1958)

4. D
 A contract whereby one person transfers the ownership of non-fungible things to another with
the obligation on the part of the latter to give things of the same kind, quantity, and quality shall
be considered a simple loan. – False (1954) BARTER
 Movable or immovable property may be the object of commodatum. – True (1937)

5. B
 If deposit with a third person is allowed, the depositary is liable for the loss if he deposited the
thing with a person. – False (1973) Only if he deposited the thing with a
person who is MANIFESTLY
 A deposit is an essentially gratuitous contract. – False (1965) CARELESS OR UNFIT
When stipulated
When the depositary is engaged
6. A in the business of storing goods
 A deposit may be constituted voluntarily or involuntarily. - True
 When it becomes necessary to open a locked box or receptacle, the depositary is presumed
authorized to do so, if the key has been delivered to him. – True (1982)

7. D When safekeeping is still the principal purpose of the contract


 When the depositary has permission to use the thing deposited, the contract loses the concept
of a deposit and becomes a loan or commodatum. – False (1978)
 If the depositary has reasonable grounds to believe that the thing has not been lawfully
acquired by the depositor, the former may return the same. – True (1984)

8. D If they are NOT solidary


 When there are two or more depositors, and the thing admits of division, each one cannot
demand more than his share. – False, 1985)

Practice Questions and Suggested Answers 73


CIVIL LAW REVIEW II

 The depositary may retain the thing in pledge until the full payment of what may be due him by
reason of the deposit. – True (1994; legal pledge)
When the thing is judicially attached
When the depositary is notified of the opposition of a
9. B third perosn to the return or removal of the thing
 The thing deposited must be returned to the depositor upon demand, even though a specified
deposited
period or time for such return may have been fixed. – False (1972, 1988)
 If the depositary by force majeure or government order loses the thing and receives another
thing in its place, he shall deliver the value of the thing lost to the depositor. – False (1990)
Sum or other thing

10. D Only true in case of GRATUITOUS DEPOSIT Death of either the depositor or depositary
 A deposit is extinguished upon the death of the depositor. – False (1995)
 The hotel-keeper has a right to retain the things brought into the hotel by the guest, as a
security for credits on account of lodging, and supplies usually furnished to hotel guests. – True
(2004; legal pledge)

11. D
 The hotel-keeper is liable for the loss of vehicles, animals and articles which have been
introduced or placed by the guests in the annexes of the hotel. – False (1999)
 A deposit is necessary when it is made in compliance with a legal obligation. – True (1996)

12. D When an attachment or seizure of property IN LITIGATION is ordered


 A judicial deposit takes place when the seizure of property is ordered. – False (2005)
 Movable property may be the object of sequestration. – True (2006)

13. D
 The act of a thief or robber, who has entered the hotel is not deemed force majeure, unless it is
done with the use of arms. – False (2001) Use of arms OR
through an irresistible force
 The death of the bailee in commodatum extinguishes the contract. – True (1939)

14. B Only when the deposit is GRATUITOUS


 A contract of deposit is a unilateral contract. - False
 A contract of deposit over movables is not covered by the Statute of Frauds. – False (ex: when
obligation is not to be performed within one year; a perfected contract is not covered because
of the doctrine of part performance) Commodatum
Bailor retains ownership
Mutuum/simple loan
15. D Ownership passes to the borrower
 In a contract of loan, the bailor retains the ownership of the thing loaned. – False (1933)
 Consumable goods may be the subject of commodatum. – True (1936)

MULTIPLE CHOICE QUESTIONS

16. A contract of commodatum (1933)


a. may be gratuitous
b. essentially gratuitous
c. may be onerous
d. essentially onerous

Practice Questions and Suggested Answers 74


CIVIL LAW REVIEW II

17. The contractual relation is known as precarium if (1947)


a. the borrower has no obligation to pay interest
b. the lender may demand the return of the thing delivered at will
c. the depositary can demand for the return at will
d. the depositary has the right to retain the thing
e. the depositary can use the thing

18. The contractual relation is known as irregular deposit if


a. the borrower has no obligation to pay interest
b. the lender may demand the return of the thing delivered at will
c. the depositary can demand for the return at will
d. the depositary has the right to retain the thing
e. the depositary can use the thing

19. This bailment is solely for the benefit of the bailor


a. mutuum
b. commodatum
c. gratuitous deposit
d. onerous deposit
e. gratuitous guaranty

20. This bailment is solely for the benefit of the bailee


a. mutuum
b. commodatum
c. gratuitous deposit
d. onerous deposit
e. gratuitous guaranty

21. In the following contract/s, ownership passes to the other party


a. mutuum
b. commodatum
c. deposit
d. a & b
e. a & c
f. a, b & c

22. The following are real contract/s


a. mutuum
b. commodatum
c. deposit
d. a & b
e. a, b & c

23. A savings deposit is


a. a contract of deposit
b. a contract of commodatum
c. an irregular deposit
d. in the nature of an irregular deposit

Practice Questions and Suggested Answers 75


CIVIL LAW REVIEW II

e. none of the above

24. The bailor cannot demand the return of the thing loaned till after the expiration of the period
stipulated, or after the accomplishment of the use for which the commodatum has been constituted,
except: (1946)
a. If the bailee devotes the thing to any purpose different from that for which it has been loaned
b. If the bailee keeps it longer than the period stipulated
c. If the bailor has urgent need for the thing
d. If the thing loaned has been delivered with an appraisal of its value
e. If he lends or leases the thing to a third person

25. The bailee in commodatum is liable for the loss of the thing, even if it should be through a fortuitous
event, except: (1942)
a. If the bailee devotes the thing to any purpose different from that for which it has been loaned
b. If the bailee keeps it longer that the period stipulated
c. If the bailee committed an act of ingratitude
d. If the thing loaned has been delivered with an appraisal of its value
e. If he lends or leases the thing to a third person

26. In commodatum, the bailor shall refund ½ of the amount spent by the bailee: (1949)
a. ordinary expense for the use of the thing
b. ordinary expense in the preservation of the thing
c. expenses for ornamentation
d. extraordinary expense for the preservation of the thing loaned arising not on the occasion of the
actual use of the thing by the bailee
e. extraordinary expense for the preservation of the thing loaned arising on the occasion of the
actual use of the thing by the bailee

27. If what was loaned is a fungible thing other than money, the debtor owes another thing of that same
kind, quantity and quality, even if it should change in value. In case it is impossible to deliver the same
kind, its value shall be determined at the time of: (1955)
a. payment
b. demand
c. the obligation becomes due and demandable
d. perfection

28. A deposit made in compliance with a legal obligation is a: (1996)


a. judicial deposit
b. extrajudicial deposit
c. voluntary deposit
d. necessary deposit

29. The depositor is obliged to reimburse the depositary for the expenses he may have incurred for the
preservation of the thing deposited if: (1992)
a. the deposit is onerous
b. the deposit is gratuitous
c. the depositary has the right to use the thing
d. the thing deposited is fungible

Practice Questions and Suggested Answers 76


CIVIL LAW REVIEW II

30. The subject matter of a contract of deposit may NOT be: (1966, 2006)
a. movable
b. immovable
c. consumable
d. non-consumable
e. non-fungible

MODIFIED TRUE OR FALSE

1. The bailee in commodatum acquires the thing loaned without compensation but not its fruits; if there
is a stipulation to the contrary, the contract ceases to be a commodatum. – FALSE
 The bailee does not acquire the thing; he acquires the use of the thing.
 Even with a stipulation, can only be for the use of the fruits, not the acquisition/ownership of
the fruits.
 Depends on what the stipulation was about:
o if it was regarding the use of the fruits: it does not cease to be a commodatum
o if it was regarding the compensation (no longer without compensation): it ceases to
be a commodatum
2. A current deposit is an irregular deposit. – FALSE
 A current deposit is a loan because the bank is allowed to use the money.
 It is only in the nature of an irregular deposit. He has aright to use and still for safekeeping.

3. In mutuum, the debtor cannot be held liable for interest if it was not expressly stipulated in writing. –
FALSE
 The debtor may still be held liable for interest when he is in delay, and there was demand
(extrajudicial or judicial)
 interest by way of damages
 compensatory interest

4. A deposit is a gratuitous contract if there is no agreement to the contrary. – FALSE


 Even if there was no agreement, it does not necessarily mean that it is gratuitous.
 It is onerous when the depositary is engaged in the business of storing goods.

ESSAY

1. Are Credit Transactions bailments?


 NOT ALL credit transactions are bailments
 must define what credit transaction and bailment, or give examples
 they are not ‘things’: they may be called principles, concepts or rules

2. On July 1, 2000, C allowed D to use his truck without compensation for a period of one year. If C
demands for the return of the truck on August 1, 2000, may it be a valid demand?
 Discuss what kind of a transaction it is; why it is a commodatum
 it may be a valid demand if:
o the bailor (C) has urgent need for the thing (1946, NCC)
o the bailee (D) has committed acts of ingratitude (1948, NCC)

Practice Questions and Suggested Answers 77


CIVIL LAW REVIEW II

3. On July 1, 1995, E borrowed money from F in the amount of P100,000.00 payable in one year. They
stipulated in writing that the loan shall bear interest at 5% per month. How much would E be liable to F
for interest as of July 1, 1996?
 The interest stipulation is void for being unconscionable, exorbitant, etc. (it is NOT the interest
that is void)
 The bailee (E) may still be held liable to pay P12,000 as interest (pursuant to legal rate)

4. A delivered a painting to B for safekeeping for a period of 6 months. If B discovered that the painting
was stolen from C, may B be relieved of all responsibilities by returning the thing to the depositor.
 The depositary (B) may be relieved if:
o he informed C (third person) of the deposit
o C did not claim the thing within the period of one month
o B returned the thing to the depositor (A) (1984, NCC)

Practice Questions and Suggested Answers 78


CIVIL LAW REVIEW II

CREDIT TRANSACTIONS

TRUE OR FALSE

1. The indivisibility of a pledge or mortgage is not affected by the fact that the debtors are not
jointly liable.
a. FALSE, debtors are not SOLIDARILY liable. (Art. 2090)
i. The rule applies if they are in fact, jointly liable.

2. Incorporeal rights may also be pledged.


a. TRUE, Art. 2095.

3. A pledge shall not take effect between the parties if a description of the thing pledged and the
date of the pledge do not appear in a public instrument.
a. FALSE, this requirement goes into the effectivity of the pledge as to third persons. The
pledge will remain valid and binding between the parties regardless of whether the
instrument is registered or not. (Art. 2096)

4. The guarantor who pays is subrogated by virtue thereof to all the rights which the creditor had
against the debtor.
a. FALSE, this statement is not absolute. An exception is when the payment by the
guarantor was made without the debtor’s consent or when the debtor already paid
when the guarantor paid. (Art. 2068-2070)

5. A guarantor may bind himself for more, but not for less than the principal debtor.
a. FALSE, a guarantor may not bind himself for more than the principal debtor; and may
bind himself for less than the principal debtor. (Art. 2054)

6. A mortgage may also be given as security for future debts, the amount of which is not yet
known.
a. TRUE. You must distinguish first –
i. A real estate mortgage, pledge, or antichresis may secure after incurred
obligations so long as these future debts are accurately described.
ii. A chattel mortgage can only cover obligations existing at the time the
mortgage is constituted.

7. The bailor in commodatum acquires the use of the thing but not its fruits; if any compensation is
to be paid by him who acquires the use, the contract ceases to be a commodatum.
a. FALSE, because it is the bailee who acquires the use of the thing, not the bailor. (Art.
1935)

8. Consumable goods may be the subject of commodatum.


a. TRUE, when the thing was borrowed not for its consumption but for a mere
exhibition. (Art. 1936)

9. Fixed, savings and current deposits of money in banks and similar institutions shall be governed
by the provisions concerning commodatum.

Practice Questions and Suggested Answers 79


CIVIL LAW REVIEW II

a. FALSE, such are governed by the provisions on simple loan, not commodatum. (Art.
1980)

10. A deposit is constituted from the moment there is a meeting of the minds upon the thing which
is the object of the contract.
a. FALSE, this is only an accepted promise to constitute a deposit. A deposit is only
perfected until the delivery of the thing. (Art. 1963)

11. Third persons who are not parties to the principal obligation may secure the latter by
mortgaging their own property.
a. TRUE, Art. 2085, last par.

12. A mortgage may also be given as security for future debts, the amount of which is not yet
known.
a. TRUE, this applies in real estate mortgage called a “dragnet clause”. This pertains to
where a mortgage is given to secure future advancements which enable the parties to
provide continuous dealings.
b. A stipulation which states that the real estate mortgage is not limited to just the fixed
amount but also covers other credit accommodations in excess thereof is a valid and
binding stipulation between the parties. (Ajax Marketing v. CA)

13. The indivisibility of a pledge or mortgage is not affected by the fact that the debtors are not
solidarily liable.
a. TRUE, Art. 2090.

14. A guaranty may be entered into without the knowledge or consent, or against the will of the
principal debtor.
a. TRUE, Art. 2050.

15. A compromise between the creditor and the principal debtor benefits the guarantor.
a. TRUE, Art. 2063.

16. The bailee in commodatum is liable for the loss of the thing, even if it should be through a
fortuitous event.
a. FALSE, there are only (5) instances when the bailee will be liable for the loss of the
thing if it should be lost through a fortuitous event –
i. Devotes the thing to any purpose different from that which it has been
loaned;
ii. Keeps it longer than the period stipulated or after accomplishment of the use;
iii. Thing was delivered with an appraisal of its value, unless there is a stipulation
to the contrary;
iv. Bailee lends or leases the thing to a third person who is not a member of his
household;
v. Ingratitude. (Art. 1942)

17. The bailor cannot demand the return of the thing loaned until after the expiration of the period
stipulated.

Practice Questions and Suggested Answers 80


CIVIL LAW REVIEW II

a. FALSE, this is the general rule. The bailor may demand the return of the thing when
there is an urgent use of the thing or when the bailee committed any act of
ingratitude. (Art. 1946 and 1948)

18. The bailor shall be liable to the bailee for the damages which the bailee may suffer by reason of
the flaws of the thing loaned.
a. FALSE, the bailor will only be liable if he knew of the flaws and did not advise the
bailee of the same and the latter suffered damages because of it. (Art. 1951)

19. Fixed, savings, and current deposits of money in banks and similar institutions shall be governed
by the provisions concerning depositum.
a. FALSE, such are governed by the provisions on simple loan, not depositum. (Art. 1980)

20. A deposit is constituted from the moment there is a meeting of the minds upon the thing
which is the object of the contract.
a. FALSE, this is only an accepted promise to constitute a deposit. A deposit is only
perfected until the delivery of the thing. (Art. 1963)

21. A guaranty may be constituted to guarantee the performance of an unenforceable contract.


a. TRUE, Art. 2052.

22. The guarantors are released from their obligation whenever by some act of the creditor they
cannot be subrogated to the rights of the latter
a. TRUE, Art. 2080.

23. A compromise between the creditor and the principal debtor benefits the guarantor.
a. TRUE, Art. 2063.

24. A guarantor may bind himself for more, but not for less than the principal debtor
a. FALSE, a guarantor may not bind himself for more than the principal debtor; and may
bind himself for less than the principal debtor. (Art. 2054)

25. If a person binds himself with the principal debtor, the contract is called a surety.
a. TRUE. By a guaranty, a person called the guarantor, binds himself to the creditor to
fulfill the obligation of the principal debtor. If a person binds himself with the principal
debtor, the contract is called a suretyship. (Art. 2047)

26. A guaranty is for compensation, unless there is a stipulation to the contrary.


a. FALSE, a guaranty is gratuitous, not for compensation, unless there is a stipulation to
the contrary. (Art. 2048)

27. If the guarantor has paid without notifying the debtor, and the latter not being aware of the
payment, repeats the payment, the former has no remedy whatever against the debtor.
a. FALSE, this is the general rule. An exception would be when the guaranty is gratuitous.
In which case, if the guarantor was prevented by a fortuitous event from advising the
debtor of the payment and the creditor becomes insolvent, the debtor shall reimburse
the guarantor for the amount paid. (Art. 2070)

Practice Questions and Suggested Answers 81


CIVIL LAW REVIEW II

28. The guarantor, even before having paid, may proceed against the principal debtor when he is
sued for payment.
a. TRUE, Art. 2071, par. 1.

29. If one, at the request of another, becomes a guarantor for the debt of a third person who is not
present, the guarantor who satisfies the debt may sue either the person so requesting or the
debtor for reimbursement.
a. TRUE, Art. 2072.

30. An obligation with a term may also be secured.


a. TRUE, “all kinds of obligations” Art. 2091.

31. The pledgor must be the absolute owner of the thing pledged when the principal obligation
becomes due and demandable.
a. FALSE, pledgor must be the absolute owner at the time of constitution of the pledge.
(Art. 2085)

32. Third persons who are not parties to the principal obligation may secure the latter by
mortgaging their own property.
a. TRUE, Art. 2085, last par.

33. A mortgage may also be given as security for future debts, the amount of which is not yet
known.
a. TRUE, you must distinguish first –
i. A real estate mortgage, pledge, or antichresis may secure after incurred
obligations so long as these future debts are accurately described.
ii. A chattel mortgage can only cover obligations existing at the time the
mortgage is constituted.

34. A pledge is indivisible.


a. FALSE, exception there being several things given in mortgage or pledge, each one of
them guarantees only a determinate portion of the credit (Art 2089, par. 4)

35. Pledge is a real contract.


a. TRUE, it is perfected by the delivery of the thing pledged. (Art. 2093)

36. Incorporeal rights may be pledged.


a. TRUE, Art. 2095.

37. A pledge shall not take effect between the parties if a description of the thing pledged and the
date of the pledge do not appear in a public instrument.
a. FALSE, pledge will remain valid as to the parties even if not in a public instrument.
However, it will not take effect against THIRD PERSONS if it does not appear in a
public instrument.

Practice Questions and Suggested Answers 82


CIVIL LAW REVIEW II

38. The ownership of the thing pledged is transmitted to the vendee or transferee as soon as the
pledgee consents to the alienation.
a. TRUE, but take note that the pledgee shall continue in possession. (Art. 2097)

39. The contract of pledge may secure all kinds of obligations, be they pure or subject to a
suspensive term.
a. FALSE, “subject to a suspensive or resolutory CONDITION” not a term.

40. The pledgee cannot deposit the thing pledged with a third person.
a. FALSE, pledgee may deposit the thing with a third person provided there is a
stipulation authorizing him to do so. (Art. 2100)

41. Deposit is a real contract.


a. TRUE, “but the deposit itself is not perfected until the delivery of the thing” (Art.
1963)

42. Deposit is not covered by the Statute of Frauds.


a. TRUE. It is the agreement to constitute a deposit which may be covered by the Statute
of Frauds. (Art. 1963)

43. Deposit by a capacitated person with another who is not, the depositor shall have action to
recover while thing is in possession of the depositary.
a. FALSE, this is not the only remedy; another is “to compel the depositary to pay him
the amount by which he may have enriched or benefitted himself with the thing or its
price. However, if a third person who acquired the thing acted in bad faith, the
depositor may bring an action against him for its recovery.” (Art. 1971)

44. The depositary is obliged to keep the thing safely and to return it to the depositor.
a. FALSE, “or to his heirs and successors, or to the person who may have been
designated in the contract” (Art. 1972)

45. If deposit with a third person is allowed, the depositary shall not be liable for the loss.
a. FALSE, the depositary is liable for the loss if he deposited the thing with a person who
is manifestly careless or unfit. (Art. 1973).

46. The surety undertakes to pay if the principal does not pay; the guarantor only binds himself to
pay if the principal cannot pay. The first is the insurer of the debt, while the other is the insurer
of the solvency.
a. TRUE, Machetti v. Hospicio de San Jose.

47. A guaranty is essentially gratuitous.

Practice Questions and Suggested Answers 83


CIVIL LAW REVIEW II

a. FALSE, unless there is a stipulation to the contrary. (Art. 2048)

48. Guaranty may be constituted to guarantee a voidable contract.


a. TRUE, Art. 2052, par. 2.

49. Guaranty may be constituted to guarantee future debts...


a. TRUE. A guaranty may also be given as security for future debts, the amount of which
is not yet known; there can be no claim against the guarantor until the debt is
liquidated. A conditional obligation may also be secured. (Art. 2053)

50. A guarantor cannot be compelled to pay the creditor unless the latter has exhausted all the
properties of the principal debtor, and…
a. FALSE, the guarantor does not have the benefit of excussion in the following instances

i. Waiver of the benefit;
ii. Liability assumed that of a surety;
iii. Insolvency of the debtor;
iv. Debtor absconds or cannot be locally sued; or
v. Resort to legal remedies is a useless formality

51. The thing deposited may not be used without the permission of the depositor.
a. FALSE, thing may be used if the use is necessary for the preservation. (Art. 1977)

52. The depositary cannot demand that the depositor prove his ownership of the thing deposited.
a. TRUE, Art. 1984.

53. The thing deposited must be returned upon demand.


a. FALSE. This is the general rule provided under Art. 1988. But this provision shall not
apply when the thing is judicially attached while in the depositary's possession, or
should he have been notified of the opposition of a third person to the return or the
removal of the thing deposited. In these cases, the depositary must immediately
inform the depositor of the attachment or opposition.

54. Deposit of effects by travellers in hotels or inns is a necessary deposit.


a. TRUE, Art. 1998.

55. Loan and deposit are essentially gratuitous.


a. FALSE, only commodatum is essentially gratuitous.

56. Bailor in commodatum acquires use of the thing loaned without compensation but not its fruits.
a. FALSE, bailee NOT bailor.

Practice Questions and Suggested Answers 84


CIVIL LAW REVIEW II

57. The bailee is not liable for the loss of the thing if due to a fortuitous event.
a. FALSE, there are only (5) instances when the bailee will be liable for the loss of the
thing if it should be lost through a fortuitous event –
i. Devotes the thing to any purpose different from that which it has been
loaned;
ii. Keeps it longer than the period stipulated or after accomplishment of the use;
iii. Thing was delivered with an appraisal of its value, unless there is a stipulation
to the contrary;
iv. Bailee lends or leases the thing to a third person who is not a member of his
household;
v. Ingratitude. (Art. 1942)

58. A contract of deposit is a consensual contract.


a. FALSE, it is a real contract.

59. An escalation clause is void if there is no de-escalation clause.


a. FALSE, Banco Filipino v. Navarro.

MULTIPLE CHOICE QUESTIONS

1. A contract which is both real and an accessory contract is:


a. Commodatum – real but not accessory;
b. Deposit – real but not accessory;
c. Mortgage – not real but accessory;
d. Pledge
e. Antichresis – not real but accessory;

In this contract, delivery is required only so that the creditor may receive the fruits.

2. A contract which is both a principal and a real contract is


a. Mutuum
b. Pledge – accessory and real
c. Mortgage – accessory and consensual
d. Antichresis – accessory and formal
e. Lease – principal and consensual

3. If a personal property is delivered to a creditor to secure the performance of an obligation, the


contract is a:
a. Pledge
b. Chattel mortgage – there is no delivery involved; personal property is merely registered;
c. Real mortgage – immovable property, not personal property is involved;
d. Antichresis – immovable property, not personal property is involved;
e. Sale

4. With this contract, the principal obligation may be extinguished


a. Pledge

Practice Questions and Suggested Answers 85


CIVIL LAW REVIEW II

b. Chattel mortgage
c. Real mortgage

In the first three choices, there must be failure of the debtor to pay the principal obligation.

d. Antichresis
e. None of the above

5. The subject matter of a contract of deposit may not be


a. Movable
b. Immovable – as a general rule, immovables may not be a subject of deposit. An exception
would be in judicial deposits.
c. Consumable
d. Non-consummable
e. Non-fungible

6. This bailment is solely for the benefit of the bailee


a. Mutuum – may also be for the benefit of the bailor when there is a stipulation for interest;
b. Commodatum – can never be for the benefit of the bailor because it will become a lease;
c. Gratuitous deposit – for the benefit of the depositor (bailor) because the deposit is free;
d. Onerous deposit – bailor and bailee benefits
e. Gratuitous guaranty – this is not a bailment

7. In the following contract/s, ownership passes to the other party:


a. Mutuum
b. Commodatum – only possession is transferred;
c. Deposit – only possession is transferred;
d. A & B
e. A & C
f. A, B & C

8. The following are real contract/s:


a. Mutuum
b. Commodatum
c. Deposit
d. A & B
e. A & C
f. A, B & C

9. A deposit made in compliance with a legal obligation is a:


a. Judicial deposit
b. Extra-judicial deposit
c. Voluntary deposit
d. Necessary deposit

10. The pledge is void if the


a. Pledgor’s consent was vitiated
b. Pledgor is suffering civil interdiction – pledgor must have free disposal of his property;

Practice Questions and Suggested Answers 86


CIVIL LAW REVIEW II

c. Pledge is not in writing


d. Pledge is not in a public instrument

11. A contract of commodatum


a. may be gratuitous
b. essentially gratuitous
c. may be onerous
d. essentially onerous

12. The contractual relation is known as precarium if:


a. the borrower has no obligation to pay interest.
b. the lender may demand the return of the thing borrowed at will.
c. the depositary can demand for the return at will.
d. the depositary has the right to retain the thing.
e. the depositary can use the thing.

13. The following are contractual bailments, except:


a. mutuum
b. commodatum
c. deposit
d. guaranty

14. This bailment is solely for the benefit of the bailor


a. mutuum – may be gratuitous or onerous
b. commodatum – essentially gratuitous
c. gratutitous deposit
d. onerous deposit – benefit for both
e. gratuitous guaranty – not a bailment

17. The depositor is obliged to reimburse the depositary for the expenses he may have incurred for the
preservation of the thing deposited if:
a. the deposit is onerous
b. the deposit is gratuitous
c. the depositary has the right to use the thing
d. the thing deposited is fungible

21. If a person binds himself solidarily with the principal debtor, he is called a:
a. guarantor
b. surety
c. solidary debtor
d. co-guarantor
e. sub-guarantor

22. The excussion may take place:


a. If the guarantor has expressly renounced it
b. If he has bound himself solidarily with the debtor
c. In case of insolvency of the debtor
d. When he cannot be sued within the Philippines

Practice Questions and Suggested Answers 87


CIVIL LAW REVIEW II

e. None of the above

23. The guarantor, even before having paid, may proceed against the principal debtor, except:
a. When he is sued for the payment
b. In case of insolvency of the creditor

Art. 2071; it is the insolvency of the debtor

c. If there are reasonable grounds to fear that the principal debtor intends to abscond
d. If the principal debtor is in imminent danger of becoming insolvent
e. None of the above

24. A guaranty may be constituted to guarantee the performance of the following obligations, except:
a. natural obligation
b. conditional obligation
c. obligation with a term
d. all of the above
e. none of the above

25. A contract of guaranty is:


a. a principal contract
b. an essentially gratuitous contract
c. a formal contract
d. all of the above
e. none of the above

27. If a person binds himself solidarily with the principal debtor, he is called a:
a. guarantor
b. surety
c. solidary debtor.
d. co-guarantor.
e. sub-guarantor.

MODIFIED TRUE OR FALSE


(Guaranty and Suretyship)

1. C
 The guaranty may be entered into without the knowledge of the principal debtor.
o TRUE. (2050, NCC)
 The guarantor may set up against the creditor all the defenses which pertain to the principal
debtor.
o FALSE. (2081, NCC)

2. C
 A guaranty may be given as security for future debts, the amount of which is not yet known.
o TRUE. (2053, NCC; continuing guaranty)
 An extension granted to the debtor by the creditor extinguishes the guaranty.
o FALSE. (2079, NCC)
Practice Questions and Suggested Answers 88
CIVIL LAW REVIEW II

3. B
 The guarantor who pays is subrogated by virtue thereof to all the rights which the creditor had
against the debtor.
o FALSE. – cannot be subrogated when the guaranty was entered into without the
knowledge of the debtor (2076, 1237, NCC)
 If the debt was for a period and the guarantor paid it before it became due, he cannot demand
reimbursement of the debtor until the expiration of the period.
o FALSE. (2069, NCC)

4. B
 If the guarantor should be convicted of a crime, the creditor may demand another who has all
the qualifications by law.
o FALSE. (2057, NCC)
 The guarantor cannot be compelled to pay the creditor unless the latter has exhausted all the
property of the debtor.
o FALSE. (2059, NCC)

5. A
 If the creditor voluntarily accepts immovable or other property in payment of the debt, even if
he should afterwards lose the same through eviction, the guarantor is released.
o TRUE. (2077, NCC; dacion en pago)
 The obligation of the guarantor is extinguished at the same time as that of the debtor.
o TRUE. (2076, NCC; because it is an accessory contract)

6. D
 The pledgor must be the absolute owner of the thing pledged when the principal obligation
becomes due and demandable.
o FALSE. (2085, NCC)
 Third persons who are not parties to the principal obligation may secure the latter by
mortgaging their own property.
o TRUE. (jurisprudence)

7. A
 A mortgage may also be given as security for future debts, the amount of which is not yet
known.
o TRUE. (has to be stipulated)
 A pledge is indivisible.
o TRUE. (2089, NCC)

8. A
 Pledge is a real contract.
o TRUE. (1313, NCC)
 Incorporeal rights may also be pledged.
o TRUE. (2095, NCC)

9. D

Practice Questions and Suggested Answers 89


CIVIL LAW REVIEW II

 A pledge shall not take effect between the parties if a description of the thing pledged and the
date of the pledge do not appear in a public instrument.
o FALSE. (2096, NCC; pledge is not a formal contract)
 The ownership of the thing pledged is transmitted to the vendee or transferee as soon as the
pledge consents to the alienation.
o TRUE. (2097, NCC)

10. C
 The contract of pledge may secure all kinds of obligations, be they pure or subject to a
suspensive term.
o TRUE. (2091, NCC)
 The pledgee cannot deposit the thing pledged with a third person.
o FALSE. (2100, NCC)

Practice Questions and Suggested Answers 90


CIVIL LAW REVIEW II

AGENCY IN GENERAL

TRUE OR FALSE

1. Notice of the revocation in a newspaper of a general circulation is a sufficient warning to third


persons.
a. TRUE, Art. 1922.

2. If two or more persons have appointed an agent for a common transaction or undertaking, they
shall be solidarily liable to the agent for all the consequences of the agency.
a. TRUE, Art. 1915.

3. If an agent acts in his own name, the principal has no right of action against the persons with
whom the agent has contracted; neither have such persons against the principal.
a. FALSE, as a general rule, the agent is the one directly bound in favor of the person
with whom he has contracted, as if the transaction were his own, except when the
contract involves things belonging to the principal. (Art. 1883, par. 2)

4. The agent may retain in pledge the things which are the object of the agency until the principal
effects the reimbursement and pays the agency for all the damages which the execution of the
agency may have caused the latter, without fault or negligence on his part.
a. FALSE, because the agent may also retain in pledge the things which are the object of
the agency not only for all the damages, but also for the sums advanced by the agent
for the execution of the same. (Art. 1912 and 1913)

5. The agent shall be liable for damages if there is a conflict between his interests and those of the
principal.
a. FALSE, because the agent will only be liable for damages, if, there being a conflict
between his interests and those of the principal, he should prefer his own. (Art. 1889)

6. Special power of attorney is necessary to loan or borrow money.


a. FALSE, the special power of attorney is not necessary to loan or borrow money IF
there is urgency or it is for the preservation of the things which are under the agent’s
administration. (Art. 1878, par. 7)

7. When a sale of a piece of an immovable property is through an agent, the authority of the latter
shall be in writing; otherwise the sale shall be void.
a. FALSE, because the law provides “sale of a piece of land”, not immovable. (Art. 1874)

8. The agency may remain in full force and effect even after the death of the agent.
a. TRUE, as a general rule, the death of the agent extinguishes the agency. However, if
the agency was constituted (a) for the benefit of both parties or (b) for the benefit of a
third person who accepted the benefit, then that agency shall continue even after the
death of the agent. (Art. 1930)

9. The principal is not liable for the expenses incurred by the agent if the agent acted in
contravention of the principal’s instructions.

Practice Questions and Suggested Answers 91


CIVIL LAW REVIEW II

a. FALSE, the principal may still be liable for the expenses incurred by the agent even if
the latter acted in contravention of the former’s instructions when the principal wants
to avail of the benefits derived by the agent. (Art. 1918)

10. By the contract of agency, a person renders some service or does something in representation
or on behalf of another, with the consent or authority of the latter.
a. TRUE, Art. 1868.
i. Alternative answer: FALSE, not all acts in representation or on behalf of
another result to agency. Art. 1868 does not contemplate social and political
representation. In order to have a contract of agency, the purpose of the
agency must be the execution of a juridical act; the act that will be executed
by the agent on behalf of the principal should create, modify, or extinguish a
legal relationship between the principal and a third person. (Uribe lecture)

MULTIPLE CHOICE QUESTIONS

1. A feature which is common to agency, negoriorum gestio and guardianship is:


a. The relationship created by law
b. Based on trust and confidence
c. There is representation
d. One has control over the other

2. The following acts may not require a SPA to be enforceable except: - malabo ung tanong at
choices. isipin mong mabuti ha.
a. to make payment
b. to waive an obligation
c. to accept an inheritance
d. to make gifts
e. to loan or borrow money – if there is urgency or if for the preservation of things which
are under the agent’s administration, the SPA will not be necessary.

3. The performance of the following acts may be delegated by a person to his agent, except:
a. to vote in a stockholders’ meeting
b. to enter into a contract of lease over his real property for more than 1 year
c. acts of strict dominion
d. to represent the principal in a meeting of board of directors’ of a company.

4. If the 3rd person with whom the agent contracted was aware that the agent was acting outside
the scope of his authority:
a. The contract is void and the principal will be liable to 3rd persons
b. the contract is void and this contract cannot be ratified
c. the contract is void and the agent will be liable to 3rd persons
d. The contract is void and the agent will be liable to the 3rd person if he undertook to
secure the principal’s ratification – Art. 1898
e. None of the above

5. A SPA may be valid even if:


a. not signed by the principal

Practice Questions and Suggested Answers 92


CIVIL LAW REVIEW II

b. not in writing
c. if is not acknowledged before a person authorized to administer oaths
d. no authority is given

6. The following are characteristics of agency as a contract:


a. consensual
b. essentially onerous
c. preparatory
d. principal

7. The agent shall not be personally liable if:


a. he acted in his own name
b. he expressly binds himself
c. he acted fraudulently
d. the exceeded his authority but the 3rd person was are of the limits of his authority

8. If the agency is couched in general terms; it comprises


a. all the business of the principal
b. only one or more transactions but not all the business of the principal
c. only acts of administration – Art. 1877
d. all acts of strict dominion

9. If the agent has been empowered to borrow money:


a. he cannot be the lender without the consent of the principal
b. he may himself be the lender at the current rate of interest
c. he may himself be the lender at the legal rate of interest
d. he may himself be the lender but the loan may comply with the Usury Law

10. Even if the agents bound themselves solidarily, they will not be held solidarily if:
a. the agent who caused damages acted in bad faith
b. the agent who caused damages acted negligently
c. the agent who caused damages acted in bad faith
d. the agent who caused damage caused beyond the scope of their authority

Practice Questions and Suggested Answers 93


CIVIL LAW REVIEW II

AGENCY

TRUE OR FALSE

1. The authority of the agent must be in writing in order for the contract that will be entered into
by the agent with a 3rd person to bind the principal.
a. FALSE, agency must be in writing only in the instances provided under Art. 1878.

2. The acceptance of the agency may be implied if the principal delivers his power of attorney to
the agent and the latter receives it without any objection.
a. FALSE, a distinction must be made if the acceptance is between persons who are
present and between persons who are absent:
i. In an acceptance between persons who are present, the statement is true.
(Art. 1871)
ii. In an acceptance between persons who are absent, acceptance cannot be
implied from the silence of the agent except –
1. Principal transmits his power of attorney to the agent who receives it
without any objection;
2. Principal entrusts to the agent by letter or telegram a power of
attorney with respect to the business in which the latter is habitually
engaged as an agent and he did not reply to the letter or telegram.
(Art. 1872)

3. When a sale of an immovable is through an agent, the authority of the latter shall be in writing;
otherwise, the sale shall be void.
a. FALSE, because the law provides “sale of a piece of land”, not immovable. (Art. 1874)

4. Agency is onerous, unless there is proof to the contrary.


a. FALSE, it is only presumed to be onerous (for compensation), unless there is proof to
the contrary. (Art. 1875)

5. A SPA is necessary to abandon a prescription already acquired.


a. TRUE, Art. 1878, par. 3.

6. A SPA is necessary to loan or borrow money.


a. FALSE, an SPA will not be required to loan or borrow money when there is urgency
and is indispensable for the preservation of the thing under administration. (Art. 1878,
par. 7)

7. The object of a contract of agency is a thing.


a. FALSE, it is the execution of a juridical act.

8. Any act which one may lawfully do personally can be delegated to an agent.
a. FALSE, there are acts that are purely personal and cannot be delegated to an agent
such as voting in national elections.

9. A person capable of acting for himself can be an agent of another.

Practice Questions and Suggested Answers 94


CIVIL LAW REVIEW II

a. FALSE, in a contract of agency, as far as third persons are concerned, what is


important and material is the legal capacity OF THE PRINCIPAL to enter into a contract.
The capacity of the agent is of no moment. The agent’s personality is merely an
extension of the principal. If the principal has the necessary capacity, that is enough.

10. The object of a contract of agency is the execution of an act in relation to a third person.
a. FALSE, must be a juridical act; not all acts in relation to ta third person are juridical
acts.

MULTIPLE CHOICE QUESTIONS

1. This is an essential requisite of a contract of agency:


a. that the agent is of age
b. that the agent shall act in representation of the principal
c. execution of a juridical act
d. the promise of the principal to pay his agent his commission

2. This act may be delegated to an agent:


a. execution of a will
b. making of an affidavit
c. to vote – take note “MAY”; a principal may ask his agent to vote in stockholder
meetings; this is a better answer than NONE OF THE ABOVE.
d. purely personal acts
e. none of the above

3. Who has the burden of proving that a contract of agency is gratuitous?


a. the principal
b. the agent
c. whoever alleges that the contract is gratuitous
d. the third person with whom the agent entered into a contract on behalf of the principal
e. no one

4. A SPA is not necessary to:


a. to accept an inheritance – (Art. 1878, par. 13)
b. to sell the car of the principal – (Art. 1878, par. 15)
c. to bind the principal in a contract of partnership – (Art. 1878, par. 10)
d. to lease the land of a 3rd person for 3 years – (Art. 1878, par. 8)
e. none of the above

5. If the agent sold an immovable property of the principal without a SPA, as a rule, the contract of
agency is:
a. void
b. voidable
c. valid – TAKE NOTE, the question is about the contract of agency, NOT THE CONTRACT
OF SALE. A contract of agency is a preparatory contract which means that there is
another contract that will be entered into.
d. unenforceable
e. rescissible

Practice Questions and Suggested Answers 95


CIVIL LAW REVIEW II

TRUE OR FALSE

1. The commission agent who does not collect the credits of his principal at the time when they
become due and demandable is liable for damages.
a. FALSE, commission agent will not be liable if he proves that he exercised due diligence
in the collection of credits.

2. Should there be a stipulation that the agent shall advance the necessary funds, he shall be
bound to do so.
a. FALSE, when the principal is insolvent, the agent will not be required to advance the
necessary funds. (Art. 1886)

3. Should the commission agent receive on a sale his commission, he shall bear the risk of
collection.
a. FALSE, this pertains to a guarantee commission. (Art. 1907)

4. If the agent has been empowered to borrow money, he may himself be the lender.
a. TRUE, Art. 1890. Take note that the agent must lend a the current rate of interest.

5. If the commission agent sold the goods on credit, without the consent of the principal, the sale
is void
a. FALSE, the sale is valid. The implication of such sale is that the principal may demand
from the agent payment in cash, but the commission agent shall be entitled to any
interest or benefit which may result from such sale. (Art. 1905)

6. The acts of the substitute appointed against the prohibition of the principal shall be void.
a. TRUE, Art. 1892.

7. Private or secret orders and instructions of the principal do not prejudice 3rd persons who have
relied upon the power of attorney or instructions shown them.
a. TRUE, Art. 1902.

8. The agent who acts as such is not personally liable to the party with whom he contracts.
a. FALSE, the agent may be personally liable when he binds himself to the third person or
he exceeds limits his authority without giving such party sufficient notice of his
powers. (Art. 1897)

9. If the agent contracts in the name of the principal, exceeding the scope of his authority, and the
principal does not ratify the contract, it shall be void.
a. FALSE, the contract may be valid. It will only be void if the third party with whom the
agent contract with is aware of the limits of the powers granted by the principal. (Art.
1898)

10. A 3rd person cannot set up the fact that the agent has exceeded his powers if the principal
ratified the agent’s acts
a. TRUE, Art. 1901.

MULTIPLE CHOICE QUESTIONS

Practice Questions and Suggested Answers 96


CIVIL LAW REVIEW II

1. If the 3rd person with whom the agent contracted was aware that the agent was acting outside
the scope of his authority:
a. Contract is void and the principal will be liable to 3rd person
b. Contract is void and this contract cannot be ratified
c. Contract is void and the agent will be liable to 3rd person
d. Contract is void and the agent will be liable to 3rd person if he undertook to secure
the principal’s ratification and he failed to do so

2. By the fact that the agent acted in excess of his authority, the contract that he entered into can
never be:
a. enforced against the principal
b. unenforceable
c. voidable
d. void

3. If a commission agent sold the goods in installment basis with interest without the authority of
the principal to sell on credit:
a. the principal will be entitled to the interest
b. the agent will not be entitled to his commission
c. the principal can demand payment from the agent in cash – Art. 1905
d. the principal can recover the things sold from the buyer

4. Even if the agents bound themselves solidarily, they will not be held solidarily if:
a. the agent who caused damage acted in good faith
b. the agent who caused damage acted negligently
c. the agent who caused damage was at fault
d. the agent who caused damage acted beyond the scope of his authority

5. If the agent has been empowered to borrow money:


a. he cannot be the lender without the consent of the principal
b. he may himself be the lender at the current rate of interest
c. he may himself be the lender at the legal rate of interest
d. he may himself be the lender but the loan must comply with the Usury Law.

TRUE OR FALSE

1. If the agent dies, his heirs must notify the principal thereof, and in the meantime adopt such
measures as the circumstances may demand in the interest of the latter.
a. TRUE, Art. 1932.

2. If the agent has exceeded his authority, the principal is jointly liable with the agent if the former
allowed the latter to act as though he had full powers.
a. FALSE, the principal is solidarily liable, not jointly, with the agent. (Art. 1911)

3. If two or more persons have appointed an agent for a common transaction or undertaking, they
shall be jointly liable to the agent for all the consequences of the agency.
a. FALSE, solidarily liable. (Art. 1915)

Practice Questions and Suggested Answers 97


CIVIL LAW REVIEW II

4. The principal is not liable for the expenses incurred by the agent when the agent incurred them
with knowledge that an unfavorable result would ensue.
a. FALSE, this will only apply if the principal was not aware that an unfavorable result
would ensue. (Art. 1918, par. 3)

5. Agency is extinguished by the death of the principal.


a. FALSE, when the agency is one coupled with interest. (Art. 1930)

6. Notice of the revocation in a newspaper of general circulation is a sufficient warning to 3rd


persons.
a. TRUE, Art. 1922.

7. The agency is revoked if the principal directly manages the business entrusted to the agent,
dealing directly with 3rd persons.
a. TRUE, Art. 1924.

8. A general power of attorney is revoked by a special one granted to another agent, as regards the
special matter involved in the latter.
a. TRUE, Art. 1926.

9. When two or more principals have granted a power of attorney for a common transaction, one
of them cannot revoke the same without the consent of the others.
a. FALSE, any one of the principals may revoke the agency without the consent of the
others. (Art. 1925)

10. An agency cannot be validly revoked if a bilateral contract depends upon it.
a. TRUE, Art. 1927.

MULTIPLE CHOICE QUESTIONS

1. The principal must reimburse agent for the sums advanced by him which shall include interest
on the sum advanced
a. from the time the agency was perfected
b. from the time the agency was made
c. from the time demand was made by the agent
d. within 30 days from demand
e. from the day on which the advance was made

2. When two persons leased the same thing, one of them with the agent and the other with the
principal, and the two contracts are incompatible with each other:
a. the contract earlier entered into shall be preferred
b. the contract entered into by the agent shall be preferred if the agent acted in good faith
c. the contract entered into by the principal shall be preferred if the principal acted in
good faith
d. the contract which was first registered in good faith

3. The principal is NOT liable for the expenses incurred by the agent in the following cases:

Practice Questions and Suggested Answers 98


CIVIL LAW REVIEW II

a. if the agent acted in contravention of the principal’s instructions


b. when the expenses were due to the fault of the principal – the law provides that the
expenses were incurred due to the fault of the AGENT, not the principal. (Art. 1918)
c. when the agent incurred them with knowledge that an unfavorable result would ensue
d. When it was stipulated that the expenses would be borne by the agent
e. All the above

4. If two or more persons have appointed an agent for a common transaction or undertaking, they
shall be:
a. solidarily liable to the agent for all the consequences of the agency
b. jointly liable to the agent for all the consequences of the agency
c. solidarily liable to the agent for all the consequences of the agency only if stipulated
d. solidarily liable to third persons

5. If the agent has exceeded his authority and the principal allowed the agent to act as though he
had full powers
a. the principal will be jointly liable with the agent
b. the principal will be solidarily liable with the agent
c. only the principal will be liable
d. only the agent will be liable

Practice Questions and Suggested Answers 99


CIVIL LAW REVIEW II

PARTNERSHIP IN GENERAL

TRUE OR FALSE

1. A partner (person) who is not a partner may not be liable as though he were a partner.
a. TRUE. This happens in a partnership by estoppel under Art. 1825 which provides that a
person who represents himself or consents to another representing him to anyone as
a partner in an existing partnership or with one or more persons not actual partner is
liable to anyone who has, on the faith of such representation, given credit to the
actual or apparent partnership.
b. Note, however, that Art. 1825 does not create a partnership as between the alleged
partners. The law only considers them as partners and the association as a partnership
insofar as it is favorable to third persons. However, the liability is created only in favor
of persons who on the faith of such representation gave credit to the partnership.

2. Every partner is a debtor of the partnership for whatever he may have promised to contribute
thereto.
a. TRUE, Art. 1786.
b. The cause of a contract of partnership is the promise of each partner to contribute
money, property or industry.

3. A partnership may be constituted in any form.


a. FALSE. This will not be true if immovable or real properties are contributed to the
partnership, in which case, the contract must be in a public instrument, recorded in
the SEC. (Art. 1771)
b. Also, if the partnership involves contributions of immovable properties, there must be
an inventory of such attached to the public instrument. Otherwise, the contract of
partnership shall be void. (Art. 1773)

4. A partner is a co-owner with his partners of specific partnership property.


a. FALSE, because this is not an ordinary co-ownership under property law. Instead, co-
ownership has specific incidence as provided under Art. 1811.
i. Alternative answer: TRUE. Express provision of Art. 1811.

5. The receipt by a person of a share of profits of a business is prima facie evidence that he is a
partner in the business.
a. FALSE. No such inference shall be drawn if such profits were received in payment:
i. As a debt by installments or otherwise;
ii. As wages of an employee or rent to a landlord;
iii. As an annuity to a widow or representative of a deceased partner;
iv. As interest on a loan; or
v. As consideration for the sale of a goodwill of a business or other property by
installments or otherwise.
b. Sharing in the profits does not necessarily result in a partnership contract because the
sharing of the profits may only be a way of compensating the other person. Such
profits may be received in payment as a debt, among others, as provided under Art.
1769, par. 4.

Practice Questions and Suggested Answers 100


CIVIL LAW REVIEW II

6. Every partnership shall operate under a firm name, which may or may not include the name of
one or more of the partners.
a. TRUE, Art. 1815.

7. Persons who are prohibited from giving each other any donation or advantage cannot enter into
a particular partnership.
a. FALSE, because the persons who are prohibited from giving each other any donation
or advantage cannot enter into a universal, not particular, partnership. (Art. 1782)

8. The contributions of a limited partner may be cash or other property, but not services.
a. TRUE, Art. 1845.

9. A partnership begins from the moment of the execution of the contract.


a. FALSE, because a partnership begins from the moment of the execution of the
contract, unless it is otherwise stipulated. (Art. 1784).

10. A person may be a general partner and a limited partner in the same partnership at the same
time.
a. TRUE, if it is stated in the certificate of limited partnership. His rights are those of a
general partner. However, with regard to his contribution, he would be considered a
limited partner, with the rights of a limited partner insofar as the other partners are
concerned.
b. Such person would have all the rights and obligations of a general partner; however,
he would have a right as to his contribution as against the other partners, which he
would not have, had he not been a limited partner. When it comes to division of
assets upon dissolution he has the priority as a limited partner.

MULTIPLE CHOICE QUESTIONS

1. This is a partner’s share of profits and surplus:


a. a partner’s rights in specific partnership property
b. a partner’s interest in the partnership (Art. 1812)
c. a partner’s right to participate in the management
d. all of the above
e. none of the above

2. The receipt by a person of a share of the profits of a business is prima facie evidence that he is a
partner in the business, but no such inference shall be drawn if such profits were received in
payment:
a. as an annuity to a widow
b. as interest on a loan
c. as the consideration for the sale of a goodwill of a business
d. all of the above (Art. 1769(4))
e. none of the above

3. This partnership comprises all that the partners may acquire by their industry or work during the
existence of the partnership:
a. universal partnership of present property

Practice Questions and Suggested Answers 101


CIVIL LAW REVIEW II

b. universal partnership of profits (Art. 1780)


c. particular partnership
d. limited partnership
e. none of the above

4. Articles of universal partnership, entered into without specification of its nature, shall be
deemed:
a. a universal partnership of present property
b. a universal partnership of profits (Art. 1781)
c. either a universal partnership of present property or a universal partnership of profits
d. neither a universal partnership of present property or a universal partnership of profits

5. Persons who are prohibited from giving each other any donation or advantage (cannot?) enter
into a:
a. universal partnership of present property
b. universal partnership of profits
c. any kind of universal partnership (Art. 1782)
d. particular partnership
e. limited partnership

6. This partnership has for its object determinate things, their use or fruits or a specific
undertaking, or the exercise of a profession or vocation:
a. universal partnership of present property
b. universal partnership of profits
c. particular partnership (Art. 1783)
d. limited partnership
e. none of the above

7. This partner cannot engage in business for himself, unless the partnership expressly permits him
to do so:
a. capitalist
b. industrial (Art. 1789)
c. limited
d. general partner
e. incoming

8. The risk of specific and determinate things contributed to the partnership so that only their use
and fruits may be for the common benefit shall, in the absence of a contrary stipulation, be
borne by the partnership except if the things:
a. are not fungible (Art. 1795) – If not fungible, the risk shall be borne by the partner
who owns them.
b. cannot be kept without deteriorating
c. were contributed to be sold
d. were appraised in the inventory
e. all of the above
f. none of the above

Practice Questions and Suggested Answers 102


CIVIL LAW REVIEW II

9. This partner cannot engage for his own account in any operation which is of the kind of business
in which the partnership is engaged, unless there is a stipulation to the contrary:
a. capitalist (Art. 1808)
b. industrial
c. limited
d. general partner
e. managing

10. Except when authorized by the other partners or unless they have abandoned the business, one
or more but less than all the partners have no authority to:
a. confess a judgment
b. submit a partnership claim or liability to arbitration
c. renounce a claim of the partnership
d. all of the above (Art. 1818)
e. none of the above

Practice Questions and Suggested Answers 103


CIVIL LAW REVIEW II

PARTNERSHIP

TRUE OR FALSE

1. A partner is co-owner with the partnership of specific partnership property.


a. FALSE, because a partner is a co-owner with the other partners, not with the
partnership, of specific partnership properties. (Art. 1811)

2. A partner cannot assign his right in specific partnership property.


a. FALSE, because in specific partnership property, while there can be no valid
assignment of rights by one partner, such assignment will be valid if all the partners
assigns their rights in the same property. (Art. 1811(2))

3. A partner’s interest in the partnership is his share of the profits and surplus.
a. TRUE, Art. 1812.

4. A stipulation which excludes one or more partners from any share in the profits is void.
a. TRUE, Art. 1799.

5. Any partner shall have the right to a formal account as to the partnership affairs if he is
wrongfully excluded from the partnership business or possession of its property by his co-
partners.
a. TRUE, Art. 1809(1).

6. Any partner shall have the right to a formal account as to partnership affairs every fiscal year.
a. FALSE. The law only enumerated the specific circumstances when a partner would
have the right to a formal account, the last of which states “whenever other
circumstances render it just and reasonable.” A partner exercising the right to a formal
account every fiscal year is not just and reasonable, considering the fact that he
already has access to the books of the partnership. (Art. 1809)

7. A partner’s right in specific partnership property is not subject to attachment or execution,


except on a claim against the partnership.
a. TRUE, Art. 1811(3).

8. A partner who has undertaken to contribute a sum of money and fails to do so becomes a
debtor for the interest and damages from the time demand is made.
a. FALSE, from the time he should have complied with his obligation. (Art. 1788)

9. The risk of specific and determinate things, which are contributed to the partnership so that
only their use and fruits may be for the common benefit, shall be borne by the partnership.
a. FALSE, because a distinction must be made as to the nature of the thing contributed:
i. If the thing contributed is non-fungible, the risk shall be borne by the partner
who owns it;
ii. If the thing contributed is fungible or it thing cannot be kept without
deteriorating or contributed by the partner to be sold, or (4) when it has an
appraised value in the inventory, the risk shall be borne by the partnership.
(Art. 1795)

Practice Questions and Suggested Answers 104


CIVIL LAW REVIEW II

10. The designation of losses and profits cannot be entrusted to one of the partners.
a. TRUE, Art. 1798, par. 2.

MULTIPLE CHOICE QUESTIONS

1. As for the profits, the industrial partner shall receive:


a. 1/3 of profits
b. equal to the share of the capitalist partners
c. in proportion to the shares of the capitalist partners
d. such share as may be just and equitable under the circumstances

2. A stipulation which excludes one or more partners from any share in the profits is known as:
a. pactum commissorium
b. pactum de non aliendo
c. pactum leonine
d. constitutum possessorium

3. The revocation of the power of a partner who has been appointed manager after the
constitution of the partnership requires:
a. just cause and the vote of the majority of the partners
b. just cause and the vote of the majority of the managing partners
c. the vote of the partners representing the controlling interest, with or without just
cause
d. the vote of the partners representing the controlling interest with just cause
e. the vote of the majority of the industrial partners, with or without just cause

4. In case it should have been stipulated that none of the managing partners shall act without the
consent of the others, the validity of the acts of each partner would require:
a. concurrence of the majority of the partners
b. concurrence of the majority of the capitalist partners
c. concurrence of the managing partners – (Art. 1802)
d. concurrence of the majority of the managing partners
e. concurrence of the partners having controlling interest

5. C, P and IP are partners, IP is an industrial partner. There is no stipulation as to the sharing of


profits and losses. The partnership realized profits of P21,000. The share of IP in the profits:
a. C and P will determine IP’s share in the profits
b. IP’s share is 7,000
c. Pro-rata to his contributed capital
d. Value of Service
e. Nothing, because he is an industrial partner

Practice Questions and Suggested Answers 105


CIVIL LAW REVIEW II

TORTS AND DAMAGES IN GENERAL

TRUE OR FALSE

1. The defense of a good father of a family is a complete and proper defense in delict.
a. FALSE, Art. 2203 provides that the party suffering loss or injury must exercise the
diligence of a good father of a family to minimize the damages resulting from the act
or omission in question.

2. An willful act may be the basis of a liability under a quasi-delict.


a. FALSE if "any willful act"; TRUE if "A willful act".

3. The liability of the employer for the acts of its employees is a subsidiary liability.
a. FALSE, an action predicated on quasi-delict means that the employer’s liability for the
negligent conduct of the subordinate is direct and primary.

4. Employers shall be liable for the damages caused by their employees and household helpers
acting within the scope of their assigned tasks, only if they are engaged in any business or
industry.
a. FALSE, employers are liable even though they are not engaged in any business or
industry. (Art. 2180, par. 4)

5. It is conclusively presumed that a driver was negligent, if he had been found guilty of reckless
driving or violating traffic regulations at least twice within the next preceeding two months.
a. FALSE, it is only a disputable presumption. (Art. 2184)

6. Respondeat superior applies to vicarious liability of employers.


a. FALSE, such doctrine holds an employer or principal principally liable for the
employee’s or agent’s wrongful acts committed within the scope of employment or
agency. In vicarious liability, only a subsidiary liability arises.

7. In motor vehicle mishaps, the owner is solidarily liable with his driver, if he was inside the
vehicle at the time of the mishap.
a. FALSE, owner must have had the opportunity, by use of due diligence, to prevent the
misfortune.

8. In quasi-delicts, the defendant shall be liable for all damages which are the natural and probable
consequences of the act or omission complained of.
a. TRUE, Art. 2202.

9. The father, and in case of his death or incapacity, the mother, are responsible for the damages
caused by their children who live in their company.

Practice Questions and Suggested Answers 106


CIVIL LAW REVIEW II

a. FALSE, minor children dapat, Art. 2180, par. 2.

10. Moderate damages, which are more than nominal but less than compensatory damages, may be
recovered when the court finds that some pecuniary loss has been suffered but its amount can
not, from the nature of the case, be provided with certainty.
a. FALSE, proved not provided, Art. 2224

MULTIPLE CHOICE QUESTIONS

1. As a source of obligations, quasi-delicts may be distinguished from the delict in that, in delict:
a. Proof requires is only preponderance of evidence
b. Right violated is a private one
c. There must be no pre-existing contractual relation between the parties
d. The defense of “good father of a family” is not a proper defense

2. The following are required to exercise utmost diligence or the highest degree of diligence,
except
a. Stevedoring companies
b. Doctors
c. Banks
d. Public utilities
e. None

3. A conclusive presumption of negligence arises when


a. An animal causes damage
b. A thing thrown from a building causes damage to another
c. The driver involved in a vehicular mishap had been found guilty of violating traffic
regulations at least twice within the next preceding two months
d. The driver, at the time of the mishap, was violating any traffic regulation
e. None

4. The degree of diligence that should ordinarily be observed in the absence of a stipulation to the
contrary is
a. Extra ordinary diligence
b. Diligence of bonus paterfamilias
c. Utmost diligence
d. Highest degree of diligence
e. None

5. The following may be held liable under quasi-delict, except


a. Minors
b. Appointive officials of the government

Practice Questions and Suggested Answers 107


CIVIL LAW REVIEW II

c. Private corporations
d. All of the above
e. None of the above

6. The liability of this person is a vicarious liability


a. Minors
b. Incapacitated persons
c. State
d. Human Resource Manage of a corporation
e. None

7. Where both parties are negligent, but the negligent act of one is appreciably later in time than
that of the other, or when it is impossible to determine whose fault or negligence should be
attributed to the incident, the one who had the last clear opportunity to avoid the impending
harm and failed to do so is chargeable with the consequences thereof. This doctrine is NOT the
a. Doctrine of supervening negligence
b. Doctrine of last clear chance
c. Doctrine of last fair chance
d. Doctrine of discovered peril
e. Doctrine of concurrent causes

8. This is only a partial defense


a. No causal relationship between the act and the damage
b. Damnum absque injuria
c. Double recovery
d. Contributory negligence
e. None

9. Proof of pecuniary loss is required for the award of


a. Lucro cessante
b. Temperate
c. Exemplary damages
d. Nominal
e. None

10. In case of death caused by a quasi-delict, the following may be awarded moral damages except:
a. Spouse
b. Father
c. Grandmother
d. Illegitimate child
e. None

Practice Questions and Suggested Answers 108


CIVIL LAW REVIEW II

TORTS AND DAMAGES

TRUE OR FALSE

1. The defense of a good father of a family is a complete and proper defense in delict.
a. FALSE. It is a defense in quasi-delict.
b. The liability of an employer when the source of the liability is culpa criminal is
subsidiary (Art. 103, RPC) and he cannot relieve himself of liability by proving due
diligence of a good father of a family. This is so because of the very nature of his
obligation. (Jurado)

2. A willful act may be the basis of a liability under quasi-delict.


a. TRUE. The concept (of a quasi-delict) covers not only acts not punishable by law but
also those punishable “whether intentional and voluntary or negligent. (Elcano v. Hill)

3. The liability of the employer for the acts of its employees is a subsidiary liability.
a. FALSE. It depends on the source of the obligation:
i. if culpa aquiliana and culpa contractual, it is direct and primary;
ii. if culpa criminal, it is subsidiary. (Jurado)

4. Employers shall be liable for the damages caused by their employees and household helpers
acting within the scope of their assigned tasks, only if they are engaged in any business or
industry.
a. FALSE. Art. 2180 provides that employers shall be liable even though they are not
engaged in any business or industry.

5. It is conclusively presumed that a driver was negligent, if he had been found guilty of reckless
driving or violating traffic regulations at least twice within the next preceding two months.
a. FALSE. The presumption is disputable. (Art. 2184)

6. Respondeat superior applies to the vicarious liability of employers.


a. FALSE.
b. The doctrine of imputed negligence or vicarious liability is not the same with
respondeat superior.
i. The doctrine of respondeat superior (“let the superior make answer”) holds an
employer or principal principally liable for the employee’s or agent’s wrongful
acts committed within the scope of employment or agency. Respondeat
superior is applicable in the liability of employers and the liability of a partner
for a tort committed by another partner. On the other hand, liability that
arises from vicarious liability is subsidiary. (Torts and Damages Reviewer, LS
)

Practice Questions and Suggested Answers 109


CIVIL LAW REVIEW II

7. In motor vehicle mishaps, the owner is solidarily liable with his driver, if he was inside the
vehicle at the time of the mishap.
a. FALSE. The owner will only be solidarily liable, if while he was in the vehicle during the
motor vehicle mishap, he could have, by the use of due diligence, prevented the
misfortune. (Art. 2184)

8. In quasi-delicts, the defendant shall be liable for all damages which are the natural and probable
consequences of the act or omission complained of.
a. TRUE. Art. 2202
i. Alternative answer: FALSE, because this only applies for actual or
compensatory damages.

9. The father and, in case of his death or incapacity, the mother, are responsible for the damages
caused by their children who live in their company.
a. FALSE. Art. 2180 provides that the father or the mother is responsible for the damages
caused by their minor children who live in their company.
b. Art. 221 of the Family Code provides that parents and other persons exercising
parental authority shall be civilly liable for the injuries and damages caused by the
acts or omissions of their unemancipated children living in their company and under
their parental authority, subject to appropriate defenses provided by law.

10. Moderate damages, which are more than nominal but less than compensatory damages, may be
recovered when the court finds that some pecuniary loss has been suffered but its amount
cannot, from the nature of the case, be provided with certainty.
a. FALSE. Art. 2224 provides that moderate damages may be recovered when the court
finds that some pecuniary loss has been suffered but its amount cannot, from the
nature of the case, be proved with certainty.

MULTIPLE CHOICE QUESTIONS

1. As a source of obligations, quasi-delict may be distinguished from delict in that, in delict:


a. Proof required is only preponderance of evidence.
b. Right violated is private one.
c. There must be no pre-existing contractual relation between the parties.
d. The defense of “good father of a family” is not a proper defense.

2. The following are required to exercise utmost diligence or the highest degree of diligence,
EXCEPT:
a. stevedoring companies
b. doctors
c. banks
d. public utilities

Practice Questions and Suggested Answers 110


CIVIL LAW REVIEW II

e. none of the above

3. A conclusive presumption of negligence arises when:


a. An animal causes damage.
b. A thing thrown from a building causes damage to another.
c. The driver involved in a vehicular mishap had been found guilty of violating traffic
regulations at least twice within the next preceding two months.
d. The driver, at the time of the mishap, was violating any traffic regulation.
e. none of the above

4. The degree of diligence that should ordinarily be observed in the absence of a law or stipulation
to the contrary is:
a. extraordinary diligence
b. diligence of bonus pater pamilias
c. utmost diligence
d. highest degree of diligence
e. none of the above

5. The following may be held liable under quasi-delict, EXCEPT:


a. minor
b. appointive officials of the government
c. private corporations
d. all of the above
e. none of the above

6. The liability of this person is a vicarious liability.


a. minors
b. incapacitated persons
c. state
d. human resource manager of a corporation
e. none of the above

7. Where both parties are negligent, but the negligent act of one is appreciably later in time than
that of the other, or when it is impossible to determine whose fault or negligence should be
attributed to the incident, the one who had the last clear opportunity to avoid the impending
harm and failed to do so is chargeable with the consequences thereof. This doctrine is NOT the:
a. doctrine of supervening negligence
b. doctrine of last clear chance
c. doctrine of last fair chance
d. doctrine of discovered peril
e. doctrine of concurrent causes

Practice Questions and Suggested Answers 111


CIVIL LAW REVIEW II

 Under the doctrine of "last clear chance" (also referred to, at times as "supervening
negligence" or as "discovered peril"), petitioner bank was indeed the culpable party.
 Stated differently, the rule would also mean that an antecedent negligence of a person
does not preclude the recovery of damages for the supervening negligence of, or bar a
defense against liability sought by another, if the latter, who had the last fair chance,
could have avoided the impending harm by the exercise of due diligence. (Philippine Bank
of Commerce v. CA)

8. This is only a partial defense.


a. No causal relationship between the act and the damage.
b. damnum absque injuria
c. double recovery
d. contributory negligence
e. none of the above

9. Proof of pecuniary loss is required for the award of:


a. lucro cessante
b. temperate
c. exemplary damages
d. nominal
e. none of the above

 Lucro cessante means lost profits.


o Indemnification for actual or compensatory damages shall comprehend not only
the value of the loss suffered but also that of the profits which the obligee failed
to obtain. (Art. 2200)
o Pecuniary loss suffered by one entitled to actual or compensatory damages must
be duly proved. (Art. 2199)

10. In case of death caused by a quasi-delict, the following may be awarded moral damages,
EXCEPT:
a. spouse
b. father
c. grandmother
d. illegitimate child
e. none of the above

Practice Questions and Suggested Answers 112

You might also like